Этого треда уже нет.
Это копия, сохраненная 15 февраля 2017 года.

Скачать тред: только с превью, с превью и прикрепленными файлами.
Второй вариант может долго скачиваться. Файлы будут только в живых или недавно утонувших тредах. Подробнее

Если вам полезен архив М.Двача, пожертвуйте на оплату сервера.
39 Кб, 500x223
153 Кб, 703x919
205 Кб, 733x906
560 Кб, 1064x792
Математика, тред 55 Теории чисел едишон Математика, тред 55 Теории чисел едишон #392366 В конец треда | Веб
Мемасы(в разработке):
http://pastebin.com/e38Yuj5V
Архивы тредов
http://pastebin.com/w1nJGYv4
#2 #392373
>>392366 (OP)
О чём пикча с говном на лопате? Пахом принёс покушать олимпиадникам?
392374
#3 #392374
>>392373
Нет, дед с dxdy просеевает числа через решето эратосфена.
513 Кб, 867x985
#4 #392387
392389
#5 #392389
>>392387
У меня встал.
392390
11 Кб, 267x354
#6 #392390
>>392389
Да у тебя и на Зорича встанет.
392396
#7 #392396
>>392390
Нет, на него не встанет, он ничего интересного не написал.
392398392459
617 Кб, 2560x1920
#8 #392398
>>392396
А на Посметьева?
392405
24 Кб, 318x417
#9 #392401
>>392366 (OP)
Сумма всех натуральных чисел равна -1/12, а число всех натуральных чисел равно -1/2. Последний факт совсем легко понять: "ясно", что число всех целых чисел -- ноль; соответственно, среди них -1/2 отрицательных, один ноль, и -1/2 положительных.
Число всех натуральных чисел -- это сумма 1+1+1+..., где единицы занумерованы натуральными числами. То есть индекс суммирования варьируется от единицы до бесконечности. Эту сумму следует отличать от суммы 1+1+1+..., где единицы занумерованы неотрицательными целыми числами (индекс суммирования варьируется от нуля до бесконечности). Вторая сумма, естественно, на единицу больше первой. Первая равна -1/2, а вторая 1/2.
Число натуральных чисел равно числу четных натуральных чисел, т.к. между ними есть естественное соответствие n <-> 2n. Следовательно, число нечетных натуральных чисел равно 0. С другой стороны, число четных натуральных чисел минус число нечетных натуральных чисел -- это просто сумма -1 + 1 - 1 + 1 - ...; она равна -1/2. Значит, четных натуральных чисел -1/2, соответственно и всех натуральных чисел -1/2.
Кстати, таким же образом можно установить, что сумма 1 + 4 + 9 + 16 + ... равна нулю. "Ясно", что сумма n^2 по всем целым n равна нулю, а сумма n^2 по всем натуральным n есть половина суммы n^2 по всем целым n.
#10 #392402
>>392401
Что за бред?
392403
#11 #392403
>>392402
https://en.wikipedia.org/wiki/Ramanujan_summation
Бред это в /b, а здесь математика.
#12 #392405
>>392398
А на этого еблана, у меня скорее ляжет.
#13 #392406
>>392401

>"ясно", что число всех целых чисел -- ноль;


Почему?
392409
#14 #392409
>>392406
Не число, а сумма. Потому что для каждого числа a есть число -a, в сумме они дают ноль.
#15 #392413
>>392401
Кстати а есть какая-нибудь интуитивная не путать с интуиционистской интерпретация всех этих петушений с суммами расходящихся рядов?
392524
#16 #392423
Анон, почему для группы G гомологии классифицирующего пространства BG с коэффициентами в G-модуле A совпадают с гомологиями, построенными как Tor_i(G, A)? И для любой ли (хорошей) группы это так?

В голову лезут только мысли про векторные расслоения, вроде теоремы Серра-Суона и идей в духе "функтор из векторных пространств в векторные расслоения представим, делаем эквивалентность категорий, смотрим на точные последовательности", но реализовать это не получается.
392428392764
#17 #392428
>>392423
А зачем ты это изучаешь? Домашка от препода в институте или есть какая-то более интересная мотивация? Ты не стесняйся, выкладывай всю подноготную, мы люди тут толерантные, плюрализм допускаем.
392431
#18 #392431
>>392428
Да у вас, батенька, параноидный психоз.
В чём может быть мотивация узнавания причин фактов? Год назад научился считать функтор Tor, полгода назад узнал за классифицирующие пространства, а сегодня прочитал статью group cohomology на Википедии, где было указано, что определения гомологий групп эквивалентны.
Стало интересно, что же от меня жидорептилоиды скрывали-то, мешая развитию руSSкой математики!
#20 #392459
>>392396
Будто Вербит что-то интересное написал.
392474
#21 #392461
>>392366 (OP)
Как вычитать бинарные числа суммированием?
Вот, есть формула: X - Y = X_2 + ¬Y_2 + 1, однако:
(10100 - 100)_10 =
(10011101110100 + 1100100)_2 =
(10011101110100 + ¬1100100 + 1)_2 =
(10011101110100 + 11011 + 1)_2 =
10011110010000 = 10128_10 ≠ 10000_10
392462
#22 #392462
>>392461

>Как вычитать бинарные числа суммированием?


Перевести в десятичные, вычесть, снова перевести в двоичные.
392463
#23 #392463
>>392462
А если его длина 256 бит и его decimal не помещается в калькулятор?
392466
#24 #392466
>>392463
В гугл-калькуляторе посчитается.
392469
#25 #392469
>>392466
Да ну тебя. Вот сам так и считай.
Помогла
$hex = unpack('H٭',$bytestring);
$big = new BigInteger('$hex', 16);
$sub = $big->subtract('$value');
$result = pack('H٭', $sub->ToHex())
Это - по числам.


А теперь вопрос: как отнять две точки на эллиптической кривой с помощью удвоений и суммирований с использованием аддитивных и мультипликативных инверсий?
392471
#26 #392471
>>392469

>php


>в там-треде


Такого у нас ещё не было.
392472392473
#27 #392472
>>392471
Хочу уметь криптить точками и декриптить байтами на элиптик курве, да.
#28 #392473
>>392471

>>в мат-треде


Фикс
#29 #392474
>>392459
Да, он написал очень много интересного.
392475392476
#30 #392475
>>392474
Много интересного хомякам говна в своем бложике. В основном о том как ебал себя в пердическое пространство объектами третьего уровня гомологичности.
392477
#31 #392476
>>392474
Из его графомании и высеров и 5% полезного не наберёться.
392477
#32 #392477
>>392475
>>392476
Как я понимаю, вы ничего кроме блога не читали.
392478
#33 #392478
>>392477
Понимаешь, что он действительно полезного написал утопает в той куче графоманского дерьма, что из себя представляют его бложики.
392479
#34 #392479
>>392478
Каким образом статьи на архиве утопают в бложике на лжр, объясни?
392480
#35 #392480
>>392479
Я говорю про всю его писанину. А не только про статьи.
392501
#36 #392481
>>392366 (OP)
Может здесь кто знает, как выразить число через сумму удвоенных единиц в различных степенях?
392482
#37 #392482
>>392481
n=1+2+4+8+...+2^k?
392485
#38 #392485
>>392482
Не, уже разобрался.
1. 123456 dec
2. в bin 11110001001000000,
3. Читать с конца
5. Если 1, то +2^N, где N - номер разряда минус 1
(1 разряд - нулевой)
6. Если ноль, то 0^N, или просто 0, или пропустить.

123456 = 2^10+0^9+0^8+2^7+0^6+0^5+0^4+2^3+2^2+2^1 = 2^10+2^7+2^3+2^2+2^1, вот.
392486
#39 #392486
>>392485

>123456 = 2^10+0^9+0^8+2^7+0^6+0^5+0^4+2^3+2^2+2^1 = 2^10+2^7+2^3+2^2+2^1, вот.


Не, и тут накосячил.
392487
#40 #392487
>>392486
123456 (10) = 11110001001000000 (2) =
2^16+2^15+2^14+2^13+0^12+0^11+0^10+2^9+0^8+0^7+2^6+0^5+0^4+0^3+0^2+0^1+0^0 =
2^16+2^15+2^14+2^13+2^9+2^6
= 65536 + 32768 + 16384 + 8192 + 512 + 64 = 123456.

Вот так будет.
9 Кб, 347x141
#41 #392494
Один анон говорил мне что интегралы нужны, например, для вычисления числа пи. Но зачем если можно просто воспользоваться пикрилейтедом? Просто задать комптьютеру каждый добавлять/минусовать/умножать следующее число и дальше будет просто вопрос времени и памяти компьютера.
Вот и всё. Никакие интегралы не нужны.
392495392496392498
#42 #392495
>>392494
> Один анон говорил мне что интегралы нужны, например, для вычисления числа пи. Но зачем если можно просто воспользоваться пикрилейтедом? Просто задать комптьютеру каждый добавлять/минусовать/умножать следующее число и дальше будет просто вопрос времени и памяти компьютера.

> Вот и всё. Никакие интегралы не нужны.


Ты ведь понимаешь, что пользуясь этими формулами ты пользуешься интегралами?
105 Кб, 558x868
111 Кб, 559x906
#43 #392496
>>392494
Вот пруф. Что они используются.
#44 #392498
>>392494
А ты не думаешь о том, что большинство из множителей - иррациональны?
392499
#45 #392499
>>392498
Просто потому, что в их знаменатели входят простые числа.
#46 #392501
>>392480
Кого ебёт его писанина за пределами архива?
392503
#47 #392503
>>392501
А ты разводишь демогогию, лол. >>392451
Ты не говорил о какой именно писанине идёт речь. Но из всей писанине статьи в архиве малая часть всего, что он написал. А написал, он чертовски много говна. Просто представь сколько времени на математику Вебритом было проёбано, когда он писал свою графоманию.
392528
#48 #392514
Утверждение а): две прямые пересекаются.
Утверждение б): две прямые параллельны.
Вопрос: какое из данных двух утверждений считается общим, а какое – частным?
392515392519392543
#49 #392515
>>392514
a) частное
б) общее
Это ведь твоя домашка, верно?
392525
#50 #392519
>>392514
Частным - значит в конкретном месте.
Общим - значит применимым ко всему.
#51 #392524
>>392413
Ну, это следствия всяких аналитических продолжений гипергеометрии, зета и тд и некоммутативности пределов. То есть, в зависимости от способа суммирования ответы могут быть разными, и интуиция не оче.
#52 #392525
>>392515
Нет, это мой интерес.
Ты ответил неверно.
#53 #392528
>>392503

>ты разводишь демогогию


Нет, демагогию разводишь ты. Речь о том, что

>он написал очень много интересного

392529
#54 #392529
>>392528

>Нет, демагогию разводишь ты. Речь о том, что


> он написал очень много интересного


А я говорю, что процент интересного от обшего процента его графомании стремится к нулю. Он так мал, что мы можем им пренебречь и считать, что Вербит не написал ничего интересного.
392531
#55 #392531
>>392529
Речь не об этом. Не о проценте годноты среди всех написанных им текстов. Речь о том, что он написал гораздо больше годноты, чем Зорич.
392533392535
#56 #392533
>>392531
Понимаешь, дерьма он написал больше. Один "Гитлер-Гитлер-Гитлер" или "Сдохнуть" чего стоит.
392542
#57 #392535
>>392531

>Не о проценте годноты среди всех написанных им текстов


>Речь о том, что он написал гораздо больше годноты, чем Зорич


Понимаешь, в процентном соотношении именно Зорич написал больше годноты.
392540
#58 #392540
>>392535
Я предлагаю засунуть тебе свои процены в анус своей мамаши, юный демагог. Потому что речь о том, что

>он написал гораздо больше годноты, чем Зорич

#59 #392541

>Секреты Математики:


>Правило 4-х знаков


Поясните.
392591
#60 #392542
>>392533
А вот это кстати годнота.
392634
#61 #392543
>>392514
Очевидно же, что 2 - частный случай первого, ибо прямые параллельны, если они пересекаются в бесконечно удалённой точке.
#63 #392594
>>392591
С последнего пика проиграл.
392596392611
#66 #392611
>>392594
А если x over 9000?
392618
#67 #392618
>>392611
Программа Посметьего его не найдёт. Она может найти корень в промежутке от -1000 до 1000.
392623
#68 #392623
>>392618
Тогда, предлагаю, забить в массив все корни от -1000 до 1000 и выискивать с помощью random. Лол.
#69 #392624
>>392607
Лакана же обоссали за то, что он неправильно её использовал.
392710
#70 #392625
>>392596
У него какая-то обида детская на математику. Будто препод по матанализу его в очко драл на сессии. Успешным он точно не выглядит.
392627
#71 #392627
>>392625
Просто он в душе ультрафинитист, пускай и слов таких не знает. Причём, радикальный! Для него существуют только натуральные числа + некоторые рациональные, которые меньше четырёх знаков после запятой.
392628
#72 #392628
>>392627
Хотя, стоп. Не все натуральные числа для него существуют. А только те, которые можно посчитать на компе.
392629392666
#73 #392629
>>392628
От минус тысячи до тысячи только.
392630
#74 #392630
>>392629
Неа, он в таких границах только квадратные корни считает.
Целые у него в диапозоне -2147483648 +2147483647.
392631
#75 #392631
>>392630
А как же длинная арифметика?
392632
#76 #392632
>>392631
Не, думаю, что ему нужны большие числа в работе. Числа, абсолютные величины которых больше 2147483647, для него это что-то мистическое.
#77 #392634
>>392542
Двачую. Читаю тифарета только из-за подобных выражений "Так ненавижу, что кушать не могу".
#78 #392636
А куда конструктивист делся? Давно его не было.
392637
#79 #392637
>>392636
Нахер ты его зовёшь? Лучше задачу предложи какую-нибудь, про ящики.
392665
#80 #392640
392680
#81 #392652
Привет, ребята!
Не подскажете ли, чему равно множество N?
392653
#82 #392653
>>392652
Числу его одноэлементных подмножеств.
392654392655
#83 #392654
>>392653
Точнее множеству, состоящему из числа его одноэлементных подмножеств. Лол.
#84 #392655
>>392653
А как определятся одноэлементные множества N?
392656
#85 #392656
>>392655
axiom of specification + axiom of pairing + axiom of power set + axiom of extensionality
#86 #392665
>>392637
Он главное в Определения N тред перекатился, а от сюда ушел. Боится уже заглядывать сюда. Хуями же накормят.
#87 #392666
>>392628
Для ультрафинитистов большие натуральные числа тоже не существуют.
392667
#88 #392667
>>392666
Вот для Посметьева и не сущесвуют.
392670
#89 #392670
>>392667
Посметьев < Ультрафинитисты < Конструктивисты < Интуиционисты < Математики < Фанаты больших кардиналов
392671
#90 #392671
>>392670
Конструктивисты < Посметьев < Ультрафинитисты < Интуиционисты < Математики < Фанаты больших кардиналов
Пофиксил.
392672
#91 #392672
>>392671
Нет, почему. Это шкала допустимых сущностей же. Ультрафинитисты радикальнее конструктивистов. Посметьев гораздо ебанутее конструктивистов.
392673
#92 #392673
>>392672

>Посметьев гораздо ебанутее конструктивистов.


Ну не знаю. Пока конструктивисты срутся на двочах со своими противниками, Посметьев гребёт бабло лопапатами, ненапрягаясь, решая всё методом перебора.
392674
#93 #392674
>>392673

>Посметьев гребёт бабло лопапатами


Поправка: он пишет во вконтакте, что гребёт бабло лопапатами.

Ну и у конструктивистов есть какая-никакая наука, а Посметьев просто ебонашка.
169 Кб, 1128x623
#94 #392676

>манятред на мейлру скатывается в обсуждение конструктивизма даже без моего участия


Лол, до сих пор печет? Ну ок, тогда еще тезисы.
1) Любая практическая математика конструктивна. Для практических целей абстракция актуальной бесконечности в принципе не нужна, т.к. все практические вычисления конечны и производятся над конечными объектами, существующими именно в конструктивном смысле. Даже если какой-то математический объект считается бесконечным, конкретные рассчеты в практическом отношении идут только с определенным количеством знаков после запятой, например.
2) Любые формулы и доказательства в формальной системе так же являются конструктивными объектами, если записываются конечным числом знаков.
3) Таким образом, настоящая неконструктивнаяч математика, с маняверой в актуальную бесконечность - это только основания, ZFC там и т.п. Но в предыдущем треде много кукарекалось, что основания это вообще не математика.
Ну и?
#95 #392677
>>392676
Ну наконец-то! Я тебя заждался уже, присаживайся, родной!
Так... На чем же остановились?
А... Ак... Акси... Аксиоматика конструктивной математики! Ты говоришь, что в конструтивной математики не аксиом, но очень многое ты принимаешь на веру. Даже не на веру, а на интуицую. Зачем ты так делаешь?
392682392711
#96 #392679
>>392676
Слушай, а как связаны алгоритмы Маркова и арифметика? Здесь что-то мелькало такое в одном из прошлых тредов.
392682
#97 #392680
>>392640
Я тебе RAGE поставил.
#98 #392682
>>392677

>но очень многое ты принимаешь на веру.


С "очень многим" уже определились, речь о критериях конструктивного объекта. Однако, это не аксиомы. Что касается веры, примеров вообще не было.
>>392679

>как связаны алгоритмы Маркова и арифметика?


Напрямую, арифметика выразима через алгорифмы Маркова.
392687392692
#99 #392687
>>392682
На чем основанные эти критерии, кроме веры?
392688392711
#100 #392688
>>392687
Я уже отвечал.
392689
#101 #392689
>>392688
Тогда дай ссылку на пост или ответь ещё раз. Правила знаешь.
#102 #392690
Я смотрю, тут просто будут копипастить все те же вопросы, игнорируя простой факт, что любая практически используемая математика конструктивна.
392691
#103 #392691
>>392690
Я бы хотел поговорить об основаниях конструктивной математики.
#104 #392692
>>392682

>Напрямую, арифметика выразима через алгорифмы Маркова.


Не, это понятно, но какая связь то? То есть вот есть арифметика, есть ном, через которые она выражается, и что? Мб у тебя какие-нить статьи есть, книги на эту тему? Я просто даже не знаю, как вопрос то сформулировать нормально.
392693
#105 #392693
>>392692

>но какая связь то?


Ну, скажем, арифметические правила - частный случай алгорифмов Маркова и любых идентичных с ними подходов (машина Тьюринга, система Поста, лямбда-исчисление Черча).

>какие-нить статьи есть, книги на эту тему?


Почти все что угодно на тему алгорифмов Маркова и всего вышеупомянутого. Как вариант http://neerc.ifmo.ru/wiki/index.php?title=Лямбда-исчисление нумералы Черча.
392694
#106 #392694
>>392693
Объясни, почему мы должны строить всю математику на алгоритмах и машине Тьюринга?
392697
#107 #392697
>>392694
Не обязательно. Только в конечном итоге все равно получится нечто вроде частного случая этого. Ну или актуальная бесконечность и во что вы там еще из неконструктивных оснований веруете. Любые конечные формулы и доказательства в любой формальной системе - кончтруктивные объекты, я жи даже цитату Лёфа принес >>392676
392700392705
#108 #392700
>>392697

>Ну или актуальная бесконечность и во что вы там еще из неконструктивных оснований веруете


Смотри, твоя конструктивная математика основана на аксиомах, так что обхывания нас верунами некоретно. Ты можешь доказать, что в твоей математики нет аксиом?
#109 #392705
>>392697
То есть подожди. Если задачу рассматривать как цепочку шагов A => B => C => ... =>Z, то только последний шаг должен быть конструктивным, так? Дозволено ли использовать неконструктивные доказательства на предшествующих шагах? Например, при доказательстве, что тот или иной алгоритм завершается. То есть разрешено ли конструктивисту использовать неконструктивные рассуждения, если они в итоге приводят к конструктивным доказательствам?
392706
#110 #392706
>>392705
Один, похоже, начал понимать, лол. У Лёфа в цитате выше ничего не сказано про обязательную конструктивность рандомной формулы. Речь только о том, что она должна быть объектом из конечного числа знаков. Тогда она будет конструктивным объектом. Конечно же, записанная в виде зависимого типа и т.п. такая формула может выдавать и жопу _|_ в качестве результата работы.

>разрешено ли конструктивисту использовать неконструктивные рассуждения, если они в итоге приводят к конструктивным доказательствам?


А будут ли такие рассуждения именно неконструктивными, а? Если итог - конструктивный объект, то очевидно, что в рассуждениях нет ничего нарушающего вычислимость.
392707
#111 #392707
>>392706
Да, допустим они будут принципиально неконструктивными. Например, в промежуточных шагах от алгоритма к алгоритму я использую ультрафильтры и аксиому выбора.
392716
#112 #392710
>>392624
Он использовал математику только для формализации своей теории и специально совершал ошибки, показывая свою "отдаленность" от дискурса математики.
мимо проходил
392712
#113 #392711
>>392676

>маняверой


>>392677

>на веру


>Даже не на веру, а на интуицую.


>>392687

>кроме веры



Математика изучает воображаемые, идеальные объекты и соотношения между ними, используя формальный язык.

Как можно вообще изучать эти воображаемые, идеальные объекты, если не принимать на веру их существование и аксиомы, позволяющие конструировать их конструктивно?
Как вообще можно изучать соотношения между ними и их системы, не принимая на веру существование критериев, на которых конструктивно основаны эти соотношения?
392714
#114 #392712
>>392710

>специально совершал ошибки


Лол. Охуительная история.
392722
#115 #392714
>>392711
Анон, ты не врубился в наш спор. Понструктивист пытается доказать, что его математика единственная правилная и что сам критерий существования нашего мира построен на ней.
И показать, что мы тупые веруны.
Я же указываю ему достойное его место - у параши. И что он тоже верит, чего он никак не хочет признавать.
392715392729
#116 #392715
>>392714

>Конструктивист


*
#117 #392716
>>392707
Тогда скорее всего вычислимость пойдет по пизде и в качестве итога работы получим жопу.
392717
#118 #392717
>>392716
Алгоритм-то получим, по условию рассматриваемой гипотетической ситуации. Неконструктивное доказательство доказывает, что алгоритм считает ровно то, что нам нужно.
392718
#119 #392718
>>392717
Тогда возникает вопрос, настолько ли это доказательство неконструктивное, как принято считать.
392719
#120 #392719
>>392718
Почему?
392720
#121 #392720
>>392719
Ну если что-то представимо в виде алгоритма, зависимого типа и не нарушает вычислимости, выдавая при проверке этого типа конструктивный объект, то почему это что-то нельзя считать конструктивным объектом?
#122 #392722
>>392712
Он даже пояснял в одном из случаев, что так высказывания не записываются в математике, но ему на это похуй. Ну и да, дискурсы то разные - учитывай.
392723
#123 #392723
>>392722
Ну ещё бы. Обосрали - начал вилять.
392724
#124 #392724
>>392723
Ей богу, кому его "обсирать" - его теорию уже критиковали другие люди (которые читали его полностью), а уж на какие-то там претензии со стороны науки он клал.
392725
#125 #392725
>>392724
Съеби из /sci.
392726
#126 #392726
>>392725
Не злись, пуся.
#127 #392729
>>392714
Дай ему Теорему Гёделя и Тарского.
392730
#128 #392730
>>392729
И проблему останова, невзначай.
#129 #392764
>>392423
Бамп вопросу. Никто что ли в мат-треде не знает гомологии групп?
392869
#130 #392869
>>392764
Лучше ты нам расскажи, по каким книгам учил Tor, Ext и Hom.
393848
#131 #392959
Ребят, где моожно найти инфу по нахождению определителя матрицы по 2ум строкам?
Пиздец, в гугле ничего не могу найти, что за бред
392979393062
#132 #392979
>>392959
Ты о чём?
392980
#133 #392980
>>392979
Знаешь, что такоое det матрицы? Ну, оппределитель
392993393006
#134 #392993
>>392980
Да, наслышан.
#135 #393003
Бля, посоны, задумался над таким вопросом
Пусть f : X -> k регулярное отображение и мы знаем, что
1) для любого x \in k x != 0 верно, что f^-1(x) неприводимо
2) Х неприводимо
хначит ли это, что слой f^-1(0) неприводим?
у кого есть идеи, как раскрутить?
#136 #393006
>>392980
полилинейная и кососиметричная штуковина, т-щ начальник!
#137 #393062
>>392959

>по двум строкам


Не понял, у тебя только две строки есть и ты по ним узнаёшь определитель или как?
#138 #393074
Посоны, как такие решать:?
-3x-12sqrt(x)=11
393077393078
#139 #393077
>>393074
Заменой sqrt(x)=t >=0. Как обычное квадратное.
#140 #393078
>>393074
sqrt(x) = x^(1/2) = y;
x = y^2 = x^(2/2) = x^1;
-3y^2 -12y = 11, значит
3y^2 + 12y + 11 = 0 - квадратное уравнение.
#141 #393095
Посоны, поясните простую вещь, почему при sqrt(x)=-1 не существует ответа? ведь и 1 и -1 в квадрате могут быть однеркой
#142 #393099
>>393095
Ты путаешь алгебраический и арифметический корень. Алгебраический корень - +. Арифметический только +. Чтобы легче считать было и убрать необдозначность корня.
https://ru.wikipedia.org/wiki/Корень_(математика)
393104
#143 #393103
>>393095
Потому что корня четной степени (sqrt это square root, т.е. корень второй степени, он же квадратный корень) из отрицательного числа в области ℝ не существует, так как при возведении в четную степень не может получиться отрицательное число (опять же, в области ℝ).
393107393108
#144 #393104
>>393095
И вот тебе ещё немного информации вдобавок к этому >>393099
У корня n-ной степени есть n возможных решений. Но выбираем мы только одно. Чтобы не было определённости.
393107393117
#145 #393107
>>393103
Но ведь не говориться что x - это отрицательное число, а говорится только что корень из x отрицательный
>>393104
Понял, спасибо, значит просто чтобы было удобно и определенно.
393108
#146 #393108
>>393107
Действительно, я прочитал твой пост жопой.
>>393103
7 Кб, 800x330
#147 #393116
>>393095

>почему при sqrt(x)=-1 не существует ответа


Ищи его в комплексной плоскости.

>ведь и 1 и -1 в квадрате могут быть однеркой


Однёрка - не минус однёрка.
393174393264
#148 #393117
>>393104

>У корня n-ной степени есть n возможных решений.


Ну, пускай n будет 4.
3^4 = 81;
4√81 = {3, -3, где остальные 2?}
3^4 = 81;
(-3)^4 = 81;
Где остальные 2?
393118393147393264
#149 #393118
>>393117
В комплексной плоскости. https://ru.wikipedia.org/wiki/Корни_из_единицы
393119393120393264
#150 #393119
>>393118
Ах, да, тогда их не n, а 2n.
393122
#151 #393120
>>393118
Хочешь сказать, что на самом деле корни имеют вид: 4√81 = {3+0i; -3+0i, 0+3i, 0-3i} ?
393121393264
#152 #393121
>>393120
Нет, я перепутал и тогда корней должно быть 8.
393123393264
51 Кб, 900x900
#153 #393122
>>393119
Т. е. это как-бы отражённый,
относительно оси Im,
пятиугольник, для 5-й степени, где одна из вершин - минус 1?
Для чётной степени он совпадает.

Да, и вот ещё...
{3+0i; -3+0i, 0+3i, 0-3i} - неправильно.
Радиус круга равен 3, значит {3+3×0i; -3+3×0i, 0+3×i, 0-3×i}
Так, что-ли?..
393141393264
#154 #393123
>>393121
Для чётной степени, многогранник совпадает сам с собой, не?
393125393264
#155 #393125
>>393123
Ну вот, например, 4-я степень...
Корень четвертой степени - это квадрат с вершинами (Re, Im):
(+1, 0), (0, +1), (-1, 0), (0, -1) - координаты их.
И одна из его вершин, третья - совпадает со значением алгебраического корня -1.
Отражая его получится тот же квадрат.
А вот пятиугольник 5-й степени не пересекает -1, если не отразить его
относительно вертикальной оси, получив 10 вершин, верно?
393264
3 Кб, 370x188
#156 #393126
Корней четвёртой степени из 1 четыре e_1,e_2,e_3,e_4
a-корень четвёртой степени из b.
Тогда (ae_i)^4=(-ae_i)^4=b*1. Ясно? Все корни четвёртой степени из единицы на пикче.
393127393129393264
#157 #393127
>>393126
Пофиксил (ae_k)^4=(-ae_k)^4=b*1
Чтобы с i путаницы не было. 1<=k<=4.
393131393264
3 Кб, 387x194
#158 #393128
393131393167393264
#159 #393129
>>393126
А последнего ответа нет, потому что ни косинус, ни синус не могут дать отрицательное. =)
У тебя там, знак умножения между i и синусом стоит?
393130393131
#160 #393130
>>393129

>У тебя там, знак умножения между i и синусом стоит?


Да.
#161 #393131
>>393129
>>393128
>>393127
Как по мне, так для четвёртой степени будут:
{1, i, -1, -i}, а не какая-то иррациональная поебень.
393132393264
#162 #393132
>>393131
Я говорю, их восемь. Ещё к + - к иррациональной поебени добавь.
393133393135
#163 #393133
>>393132
У тебя что, восьмиугольник?
393134
#164 #393134
>>393133
Я не стараюсь их визуализировать, но скорее получаются два четырёхугольника, а не восьмиугольник.
393136
#165 #393135
>>393132
Если восьмиугольник, то + косинус и - косинус запрогь себе.
393140
#166 #393136
>>393134
ИМХО, они совпадают.
А если нет, то может в одном из слагаемых числитель со знаменателем надо менять?
9 Кб, 903x611
#167 #393138
Как-то так все корни расположены. Нарисовал криво, но это дожен быть звёдчатый восьмиугольник.
393139393141393264
#168 #393139
>>393138
Хотя, нет! Он должен выть вогнут во внутрь. Я сонный, и рисовав его посчитал, что sqrt(2)/2>1. Сейчас перерисую.
393141
#169 #393140
>>393135

>- косинус


-синус, хотел сказать

>может


> числитель со знаменателем надо менять


Чтоб было вот так:
e1 = cos((пи×0)/4) + i×sin((пи×0)/4);
e1 = -cos((пи×0)/4) + i×sin((пи×0)/4);
e1 = cos((пи×0)/4) - i×sin(4/(пи×4));
e1 = -cos((пи×0)/4) - i×sin(4/(пи×4)); -кажется тут вариантов уже 16
e1 = -cos((пи×0)/4) + i×sin(4/(пи×4)); -а тут 32
393143
#170 #393141
>>393138
>>393139
Ну, смотри, он вписан в круг >>393122
Поверни многоугольник относительно вертикальной оси и получишь эти свои 10 вершин.
А потом, можель повернуть и относительно диагональных осей.
15 Кб, 800x600
#171 #393142
Вот как-то так.
393166393264
#172 #393143
>>393140
Что это за бред? Ты видимо дохуя всего напутал.
393146
107 Кб, 1046x607
150 Кб, 1052x573
132 Кб, 1069x527
#173 #393144
По поводу аксиом в конструктивной математике. Речь о том, что их там нет в неконструктивном смысле. Т.е. в том смысле, что в них надобно веровать. В конструктивной математике аксиома - это всего лишь формула, секвенция и т.п. без посылок. Такую аксиому можно использовать в построении объектов, если она не нарушает вычислимости, в противном случае аксиома бесполезна, хоть веруй в нее, хоть нет. Пример - пикрелейтед1, не считая ошибки - там должно быть r2, а не r5. Т.е. в конструктивной математике в конечном счете любая т.н. аксиома - это теорема, которую можно доказать либо свести к абсурдности. Даже эта ваша аксиома выбора, в которую вы веруете, для некоторых случаев может быть доказана конструктивно, пикрелейтед2. Разница между конструктивной и неконструктивной логикой была показана Мартин-Лёфом, пикрелейтед3. Суть в том, что эта ваша неконструктивная логика соответствует синтетическому суждению по Канту, она неполна и неразрешима по Геделю. Тогда как конструктивный подход в логике соответствует кантовскому аналитическому суждению и т.о. конструктивная логика полна и разрешима.
#174 #393145
>>393144
Пошел нахуй отсюда, мы тут комплексные числа обсуждаем.
#175 #393146
>>393143
А ты подставь и возведи в 4-ю. Циклом.
393148
#176 #393147
>>393117
Добро пожаловать на комплексную плоскость.
#177 #393148
>>393146
Анон, у тебя проблеммы с тригонометрией. Поскольку в твоих формулах
e1 = cos((пи×0)/4) + i×sin((пи×0)/4);
e1 = -cos((пи×0)/4) + i×sin((пи×0)/4);
e1 = cos((пи×0)/4) - i×sin(4/(пи×4));
e1 = -cos((пи×0)/4) - i×sin(4/(пи×4)); -кажется тут вариантов уже 16
e1 = -cos((пи×0)/4) + i×sin(4/(пи×4)); -а тут 32

Только два числа +1 -1.
Это получается из cos((пи×0)/4)=1 sin((пи×0)/4)=0.
Серьёзно, ты поебень написал. Да и если cos((пи×k)/n) + i×sin((пи×k)/n) - корень, не факт что все остальные тоже.
393150393264
#178 #393149
Вот ещё, что если n -нечётное, то алгебраических корней из единицы будет только n. Т.к. (-1)^n=(-1)
Если n-четное, то 2n. Т.к. (-1)^n=(1)^n=1
#179 #393150
>>393148

>корень, не факт что все остальные тоже


В смысле, те которые ты написал:
e1 = cos((пи×0)/4) + i×sin((пи×0)/4);
e1 = -cos((пи×0)/4) + i×sin((пи×0)/4);
e1 = cos((пи×0)/4) - i×sin(4/(пи×4));
e1 = -cos((пи×0)/4) - i×sin(4/(пи×4)); -кажется тут вариантов уже 16
e1 = -cos((пи×0)/4) + i×sin(4/(пи×4)); -а тут 32
393161393264
#180 #393151
>>393144
Откуда берутся правила построение? И откуда берутся формулы?
393153
#181 #393153
>>393151

>Откуда берутся правила построение? И откуда берутся формулы?


Из постановки задачи, которую требуется решить.
393154
#182 #393154
>>393153
А откуда берутся задачи и формулы?
393157
#183 #393157
>>393154
Из целесообразно-извлечённой информации.
393158
#184 #393158
>>393157
Откуда она извлекается?
393159
#185 #393159
>>393158
Из окружающей среды.
393160
#186 #393160
>>393159
А каким образом?
393163
#187 #393161
>>393150
Ну, это я от балды.
Ты же прогишь там - так и проверь заодно...
А ещё, вот, угол ты какой берёшь?
Пи. Относительно каких на окружности?
Могут ли углы, входящие в слагаемые косинусы и синусы изменяться при расчёте корня?
Если да, то симметричны ли эти углы?
Если симметричны, то относительно каких осей?

Может ли, например,
cos(пи×1/4) + i×sin(пи×3/4) давать корень 4-й степени в комплексной плоскости?
а -cos(пи×4/4) + i×sin(пи×1/4)?
В общем, ещё и углы можно пошатать, вдруг там комплексные корни найдутся?
393165393264
#188 #393163
>>393160
Матрично-алгоритмическим.
#189 #393165
>>393161

>А ещё, вот, угол ты какой берёшь?


Из формулы муавра.
https://ru.wikipedia.org/wiki/Корни_из_единицы

>Относительно каких на окружности?


Окружность с центром в начале координат.

>Могут ли углы, входящие в слагаемые косинусы и синусы изменяться при расчёте корня?


Формула Муавра. Там всё написанно.

>Если да, то симметричны ли эти углы?


Относительно комплексного сопряженного.
393169393264
#190 #393166
>>393142

Блядь. Я в той пикче двойку забыл перед пи поставить. Пойду спать. Сука.
393167
#191 #393167
#192 #393169
>>393165

>Относительно каких на окружности, имелось в виду.


точек, имелось в виду.

>Окружность с центром в начале координат.


И радиусом, равным длине от начала координат до значения подкоренного выражения.
Это понятно.

>Относительно комплексного сопряженного.


z = a + bi; z = a - bi - комплексно-сопряженное, по определению.

Т. е. ты отражаешь вторую точку на окружности относительно горизонтальной оси координат?
А почему бы её не отразить относительно вертикальной?
z = a + bi; z = -a + bi
Или ваще, так: z = a + bi; z = -a - bi? Лол.
393173
#193 #393173
>>393169
Если тебе нужно поделить 2 компл. числа: (a+bi)/(c+di), то непонятно, как это делать.
Если числ. и знаменатель домножить на одно и то же число, то значение дроби не изменится. Если домножить на сопряженное к (c+di), то в знаменателе появится действительное число[(c+di)(c-di) = (c^2+d^2)], а поделить комплексное на действительное уже можно.
Поэтому вот так вот сопряжение определяется.
393264
#194 #393174
>>393116
.gif
найс скример, браток
393181
#195 #393179
Хочу читать 700 страниц в час.
Какие подводные?
393182393193393196
131 Кб, 800x330
#196 #393181
>>393174
На, раз уж ты так сильно хочешь...
393210
#197 #393182
>>393179
Твоя мамаша.
#198 #393193
>>393179

>Какие подводные?


Это сложный вопрос, но подводные...

___КАМНИ___
___А________
___М________
___Н________
___И________
#199 #393196
>>393179
"Правда, такая ерунда получается".
#200 #393210
>>393181
За запись i=sqrt(-1) нужно по яйцам бить например.
393260
#201 #393211
>>393144
В классической математике это тоже всего лишь формула.
#202 #393260
>>393210
Эта пикча с википедии. Но ты прав, антуан.
i^2 = -1, и на самом деле -1 = sqrt(i)
393261
#203 #393261
>>393260
Даже не с википедии, а отсюда: http://refy.ru/images/56/1394905313_116.gif
#204 #393264
>>393116
>>393117
>>393118
>>393120
>>393121

>С чего это 8?


>>393122 >>393165
Из этих формул вытекает, что корней из единицы всегда ровно n, и все они различны.
>>393122 >>393125
-1^5 не даст 1, поэтому отражать не нужно. Это же корни из 1, а не из (-1).
Степень - нечётная.
>>393123
Да.

>>393131
4√1 = {1, -1, i, -i};
1^4 = 1;
(-1)^4 = (-1)×(-1)×(-1)×(-1) = 1×1 = 1;
i^4 = i×i×i×i = i^2 × i^2 = (-1)×(-1) = 1;
(-i)^4 = (-i)×(-i)×(-i)×(-i) = i^2 × i^2 = (-1)×(-1) = 1;
Вот это - проверить можно.
А остальное, >>393126>>393127>>393128>>393148>>393150>>393161
как проверить?

>>393138>>393142
Все точки лежат на окружности, радиусом 1, для корня n-й степени из 1.

>>393173
А зачем делить?
#204 #393264
>>393116
>>393117
>>393118
>>393120
>>393121

>С чего это 8?


>>393122 >>393165
Из этих формул вытекает, что корней из единицы всегда ровно n, и все они различны.
>>393122 >>393125
-1^5 не даст 1, поэтому отражать не нужно. Это же корни из 1, а не из (-1).
Степень - нечётная.
>>393123
Да.

>>393131
4√1 = {1, -1, i, -i};
1^4 = 1;
(-1)^4 = (-1)×(-1)×(-1)×(-1) = 1×1 = 1;
i^4 = i×i×i×i = i^2 × i^2 = (-1)×(-1) = 1;
(-i)^4 = (-i)×(-i)×(-i)×(-i) = i^2 × i^2 = (-1)×(-1) = 1;
Вот это - проверить можно.
А остальное, >>393126>>393127>>393128>>393148>>393150>>393161
как проверить?

>>393138>>393142
Все точки лежат на окружности, радиусом 1, для корня n-й степени из 1.

>>393173
А зачем делить?
393403
#205 #393277
Параллельные прямые пересекутся когда-нибудь?
393278
#206 #393278
>>393277
Слова "две прямые параллельны" значат "две прямые не пересекаются". Прямые, которые не пересекаются, - не пересекаются.
393279
#207 #393279
>>393278
А в бесконечности перекутся?
393280393293
#208 #393280
>>393279
Констурктивиста сейчас небось триггернуло от твоего вопроса. Лол.
393281
#209 #393281
>>393280
Пересекутся или нет?
393282
#210 #393282
>>393281
Хуй себе пересеки.
393288
#211 #393288
>>393282
Выходит, пересекутся.
393292
#213 #393293
>>393279
В не Евклидовой геометрии, возможно, да - в зависимости от кривизны пространства.
#214 #393304
ОП, где шапка? Почему с вопросом о корнях приходят сюда, а не в тред для начинающих?
393308
# OP #215 #393308
>>393304

>ОП, где шапка?


Постоянной шапки для этого треда никогда и не было. Надо бы сделать.

>Почему с вопросом о корнях приходят сюда, а не в тред для начинающих?


Аноны с самого начала путают треды. Как с этим бороться не знаю. Да и нужно ли? Может две ветви снова в одну объединить?
393309
#216 #393309
>>393308
А, я здесь с тредов, когда в шапке было типа - обсуждение листков НМУ; новички идите в тред для начинающих.
Вообще да, мне кажется хорошо бы объединить. В шапку накидать списки, рекомендации, архив мемасов и сказку какую-нибудь.
393314
# OP #217 #393314
>>393309

>А, я здесь с тредов, когда в шапке было типа - обсуждение листков НМУ; новички идите в тред для начинающих.


Так и было. Работало не сильно, как помню.
Два треда протекали в друг-друга(да и сейчас так делают), а потом и вовсе аноны перебрались в тред для начинающих. Обычный мат-тред почти умер. Возможно, сейчас он жив только потому что я не стал писать это предупреждение.
393327
#218 #393327
>>393314
Нужно объединить тогда.
393332
# OP #219 #393332
>>393327
Тогда вопрос как именно? Можно продолжить одну из ветвей и закрыть другую. А можно закрыть обе и начать новую ветвь "Объединенный мат-тред".
393359
#220 #393333
Верно ли следующее доказательство обобщённого закона ассоциативности?
Пусть G - группа.
Требуется доказать, что произведение g1 g2 ... gn не зависит от расстановки скобок.
Док-во:
по индукции. база n=3 - верно (аксиома)
предположим, что верно для n = k,
докажем, что верно для n = k+1
(g1 g2 ... gk)gk+1 = ((g1 g2...gm)(gm+1...gk))g_k+1
Пусть g1 g2...gm = a
gm+1...gk = b
Тогда (ab)g k+1 = a(b*g) в силу аксиомы
Верно?
28 Кб, 726x445
#221 #393338
Тред определения N закрыли, поэтому определять будем тута. На сей раз хотелось бы услышать внятных аргументов, почему нумералы Черча не определяют N. https://www.r-bloggers.com/church-numerals-in-r-or-how-to-prove-the-existence-of-natural-numbers-using-the-lambda-calculus/ ведь все построимо.
393339393340
#222 #393339
>>393338
Есть аксиомы пеано. Точка.
393341393344
#223 #393340
>>393338

Потому что ты не можешь юзать индукцию до её определения.
393344
#224 #393341
>>393339

Аксиомы которые не являются определением N.
393343
#225 #393343
>>393341
Это ещё почему?
#226 #393344
>>393339
Они базируются на уже готовых числах N и сами натуральные числа не определяют.
>>393340

>ты не можешь юзать индукцию до её определения.


Я ее построить могу. Это и есть определение. Определение индукции, оторванное от самой индукции - это мистика.
393346
#227 #393346
>>393344
А верить в правила построения не мистика?
393347
#228 #393347
>>393346
А я в них не верю, я их применяю. Верой было бы нечто оторванное от самих правил построения.
393348
#229 #393348
>>393347

>https://ru.wikipedia.org/wiki/Определение_(логика)


Что-то я тут слова построить не вижу. А про аксиомы вижу.
393350
#230 #393349
Как ты докажешь, что правила построения это и есть определение?
393350
#231 #393350
>>393349
Их применение дает в итоге конструктивный объект - N.
>>393348
Неконструктивная шляпа.
393351
#232 #393351
>>393350

>Неконструктивная шляпа.


А я могу сказать, что конструктивизм - дерьмо. Ты по существу что-то сказать можешь?

>Их применение дает в итоге конструктивный объект - N.


Я задал вопрос не про N, а в общих чертах. Как ты докажешь, что правила построения и есть определение?
393353
#233 #393353
>>393351

>Ты по существу что-то сказать можешь?


Могу. Логика - часть математики. Она может применяться как часть математики, но не может служить обоснованием математики.

>Я задал вопрос не про N, а в общих чертах. Как ты докажешь, что правила построения и есть определение?


Разницы нет, в общих или не в общих. Правила построения определяют объект непосредственно его построением, в результате которого и получаем построенный объект, который т.о. исчерпывающе определяется правилами своего построения.
#234 #393354
>>393353

>Могу. Логика - часть математики. Она может применяться как часть математики, но не может служить обоснованием математики.


Почему? А грамматика слово "грамматика" описывать может?
393356
#235 #393355
>>393353
А что такое правила построения и что такое объект?
393356
#236 #393356
>>393354

>А грамматика слово "грамматика" описывать может?


Может. Но в отрыве от грамматических правил это просто слово.
>>393355
Ой, не надо по новой. Каждый тред одно и то же. Будто я не писал что такое конструктивный объект, не писал про системы Поста, алгорифмы Маркова, лямбда-исчисление Черча и протчая и протчая. Мне в каждом треде с этого начинать?
393363
#237 #393357
>>393353

>Логика - часть математики.


Смелое заявление. На чём оно основывается?

>Она может применяться как часть математики, но не может служить обоснованием математики.


Почему нет? Разве ты в своей конструктивной параше разве не исользуешь логику для описания правил построения?

>Разницы нет, в общих или не в общих.


Есть.

>Правила построения определяют объект непосредственно его построением, в результате которого и получаем построенный объект, который т.о. исчерпывающе определяется правилами своего построения.


Ты доказал, только что из правил построение можно что-то построить. Всё. Это не определение. Например, строитель может построить дом, у него есть правила построения(четрём там все дела). Но он может ничего не знать о молекулах, атомах, кварков и т.д, тех материалов, которые использует при строительстве. Ему не нужно определения того, с чем он работает. Так же и ты.
#238 #393358
>>393357

>четрёж


*
#239 #393359
>>393332
Не знаю. Я здесь не один. Спроси у ОПа треда для начинающих; спроси у анончиков обоих тредов; проведите голосование. Может они и объединяться не захотят.
393360
# OP #240 #393360
>>393359
Это я и есть. Два треда веду.
393362
#241 #393361
>>393357
Да на все это сто раз отвечено, трюле. Я даже в этом треде >>393144 уже пояснял. Принес вот нумералы Черча, а все опять скатывается к бесконечному повторению азов.
393364
#242 #393362
>>393360
Не знаю в общем, решай сам, ТЫЖЕОП.
#243 #393363
>>393356
Я просто не понимаю что значит "дать основания", по мне так к этому многие люди приплетают какую-то ненужную метафизику. По мне так "дать основания" значит свести математику к каким-то более простым и более понятным любому человеку концепциям. Классическая математика (формализм типа Гильберта) сводит к "теории первого порядка" и в ней уже "правилам вывода" одних строк из других; конструктивизм типа Маркова сводит к алгоритмам - неважно в какой формализации - через нумералы Чёрча или через программы на pure C. В упор не вижу по каким причинам один подход это "основания" а другой "не основания.
#244 #393364
>>393361
Я спрашиваю у тебя другие вопросы. Перечитай ещё раз где в моём посте >>393357 есть что-то про аксиомы. С ними мы разобрались и сейчас я задаю новые вопросы. Отвечай на них. Не увиливай.
#245 #393367
>>393357

> Разве ты в своей конструктивной параше разве не исользуешь логику для описания правил построения?


Есть логика и логика. Я же специально >>393144 пояснил в чем разница, третий пик. Неконструктивная логика либо неполна, либо противоречива по Геделю, там написано почему так.

>Ты доказал, только что из правил построение можно что-то построить. Всё. Это не определение. Например, строитель может построить дом, у него есть правила построения(четрём там все дела). Но он может ничего не знать о молекулах, атомах, кварков и т.д, тех материалов, которые использует при строительстве. Ему не нужно определения того, с чем он работает. Так же и ты.


Будто для того чтобы построить дом, нужно знать за каждый кварк в стройматериалах. Дом будет построен и без этого. А кварками ядерная физика занимается, а не строители домов. Какая задача, такое и решение.
393369
27 Кб, 622x136
#246 #393369
>>393367

>Неконструктивная логика либо неполна, либо противоречива по Геделю, там написано почему так.


Пик.

>Будто для того чтобы построить дом, нужно знать за каждый кварк в стройматериалах. Дом будет построен и без этого. А кварками ядерная физика занимается, а не строители домов. Какая задача, такое и решение.


Я рад, что ты понимаешь, что правило построение - не определения объекта. Мы пришли к согласию?
393371
#247 #393371
>>393369
Генцена тоже разбирали уже, у него там конструктивизм на конструктивизме едет и конструктивизмом погоняет, потому у него и получилось доказать непротиворечивость арифметики. На одном формализме до сих пор бы не доказали что 1+1=2. Построение - это определение объекта, я уже сто раз объяснял почему.
393374393413
#248 #393372
Проходил мимо и решил заглянуть: N-петух в конструктивизм-петуха переквалифицировался или это разные персонажи?
#249 #393374
>>393371

>Построение - это определение объекта, я уже сто раз объяснял почему


Ты пытался объснить, а не доказать. Я хочу доказательство этого. Пример с домом опроверг твоё доказательство здесь >>393353

>Генцена тоже разбирали уже, у него там конструктивизм на конструктивизме едет и конструктивизмом погоняет


Он ученик Гилберта, а не Брауэра. Зачем ученику Гилберта заниматься констркутивной математикой?
393375
#250 #393375
>>393374

>Зачем ученику Гилберта заниматься констркутивной математикой?


А Гильберту зачем? Метаматематика -конструктивный подход. Как и почти вся остальная математика, опять же я объяснял почему и даже Мартин-Лёфа цитировал. Говорю же, обсуждалось все это.

>Пример с домом опроверг твоё доказательство


Ну раз ты так сказал, то ладно. Даже лень объяснять, почему дом это не аналогия с конструктивными математическими объектами, настолько все очевидно и однозначно.
393379
#251 #393376
Вот тебе ещё в догонку.
Ты говоришь, что определение объекта - правила его построения.
Тогда какие правила построения у фотона, например? Как ты будешь строить фотон? Или какие правила построения у бозона Хигса? Если их нет, то эти элементарные частицы неопределены и значит физика сплошное вернуство?
393378
34 Кб, 678x746
#252 #393378
>>393376
Ты не очень умный, да? Мы про математику и про математические объекты, а не про кобзон Хиггса.
393380
#253 #393379
>>393375

>настолько все очевидно и однозначно


>очевидно


Обычно, когда пишут очевидно в мматематике, то всё не так уж и очевидно. Ты не ленился писать "Брауэр" несколько сотен раз, а написать доказательство лень.

> лень объяснять


Мне нужно ДОКАЗАТЕЛЬСТВО, а не ОБЪЯСНЕНИЯ! Ты видишь между этими понятиями разницу?!
393383
#254 #393380
>>393378
Если математические объекты не имеют ничего общего с реальностью, то почему бесконечность не может существовать в математике? Ведь приминение твоих констурктивных приёмчиков к реальности полностью их дискредитирует.
393383
#255 #393383
>>393379

>Мне нужно ДОКАЗАТЕЛЬСТВО, а не ОБЪЯСНЕНИЯ! Ты видишь между этими понятиями разницу?!


А ты видишь разницу между математическим объектом и фотонами и бозонами? Какие могут быть математические правила построения у элементарных частиц, совсем уже школота тутошняя поехала.
>>393380
И на это отвечал. Существование в математике = построение. Существование в математике чего-то, не сводящееся к построению этого чего-то = мистика. Что значит "существует бесконечность", объясни? И чем такое "существование" отличается от существования Аллаха?
393384393386393435
#256 #393384
>>393383

>Что значит "существует бесконечность", объясни?


Значит, что соответствующая формула выводится из аксиом ZFC.
393385
246 Кб, 736x732
#257 #393385
>>393384

>Значит, что соответствующая формула выводится из аксиом ZFC.


А из корана выводится существование Аллаха. Чем твоя вера в KFC ZFC лучше? В обоих случаях какие-то там аксиомы, не дающие возможности построить объект, "определяемый" ими. Впрочем, это тоже уже обсуждалось.
#258 #393386
>>393383

>А ты видишь разницу между математическим объектом и фотонами и бозонами? К


Стоп.

>Существование в математике = построение.


Ну если так сказал анон с мейл.ру, то да.
http://ponjatija.ru/node/800 Тут другие определения существования.

>А ты видишь разницу между математическим объектом и фотонами и бозонами? Какие могут быть математические правила построения у элементарных частиц, совсем уже школота тутошняя поехала.


То есть мы обсуждаем не реальный мир а абстрации? Что-то вроде платоновского мира, так. Где объекты существуют сами по себе, оторванные от реальности.

>Существование в математике чего-то, не сводящееся к построению этого чего-то = мистика.


Раз уж мы оторвались от реального физического мира, то какое это имеет значение? И как же непротиворечимость теории? Мы всеравно говорим об астрациях, причём твои правила к примению в физическом мире приводят к породоксам похлеще удвоения шара.
393387
#259 #393387
>>393386
Все это тоже было уже. Правда, мне лень по десять раз писать одно и то же. На сей раз я пришел с конкретной темой - определение N через нумералы Черча.
393388393414
#260 #393388
И ещё ты говорил, что логика - часть математики. Тогда я только, что нашел в ней противоречие, логика говорит нам, что фотоны существуют, констуктивная математика говорит нам, что фотоны не существуют, т.к нет правил их построяния. Значит либо физика ложна, либо констктивнмая математика. Если конструктивная математика ложна, то и логика ложна (ты сказал, что логика не использутся для констктивной математики, но констуктивная математика используеться для логики).
Физика не может быть ложной, значи ложна конструктивная математика, а значит и логика, значит, только, что выведенное ложно. Если оно ложно, то констуквтиная математика истинна, если она истинна, то высказывание ложно. Парадокс. Значит, логика - не часть математики.

>>393387
Не ври. Про физику я только сейчас сказал. Это новая тема.
393390
howto отвечать как конструктивис #261 #393389
— Почему существует только то, что порождается алгоритмом?
— Потому.

— Ты ебанутый?
— На это неоднократно давался ответ.

— Ты можешь доказать формулу Гаусса?
— Твоя вера не нужна.

— Пошёл нахуй, ебанашка.
— Ты не воспринимаешь объективные аргументы.

— Какие?
— Хватит спрашивать одно и то же.

— Ты издеваешься. Ты не можешь писать это всерьёз.
— Я не виноват, что ты такой тупой.
1733 Кб, 200x200
#262 #393390
>>393388

>Значит либо физика ложна, либо констктивнмая математика.


Честно скажу, на этот бред кроме пикрелейтед даже не знаю что ответить. Вся суть школьной манялогики.
393391
#263 #393391
>>393390

>ты сказал, что логика не использутся для констктивной математики, но констуктивная математика используеться для логики


Твои цытатки?

>Могу. Логика - часть математики. Она может применяться как часть математики, но не может служить обоснованием математики.


>констуктивная математика - основания математики


Просто признайся, что ты обосрался.
#264 #393392
бля ну скажите плз дауну заебали.

Верно ли следующее доказательство обобщённого закона ассоциативности?
Пусть G - группа.
Требуется доказать, что произведение g1 g2 ... gn не зависит от расстановки скобок.
Док-во:
по индукции. база n=3 - верно (аксиома)
предположим, что верно для n = k,
докажем, что верно для n = k+1
(g1 g2 ... gk)gk+1 = ((g1 g2...gm)(gm+1...gk))g_k+1
Пусть g1 g2...gm = a
gm+1...gk = b
Тогда (ab)g k+1 = a(b*g) в силу аксиомы
Верно?
393393393396393405
#265 #393393
>>393392
Я бы проверил, но до сих пор не врубаюсь в доказательство этой теоремы. И я просто забил на неё вообще. Можешь сверить своё доказательство с доказательством и з Теорим Абеля в Задачах.
393405
#266 #393396
>>393392
Можешь посмотреть у Кострикина доказательство.

>(g1 g2 ... gk)gk+1 = ((g1 g2...gm)(gm+1...gk))g_k+1


С этим соглашусь.

>Тогда (ab)g k+1 = a(b*g) в силу аксиомы


С этим тоже.

Каким образом из этого следует, что в изначальном произведении можно выбирать любую расстановку скобок?
393465393569
#267 #393403
>>393264
>>392366 (OP)

Ну, вот, взять например,
5√1;
Пускай это будет cos(2П4/5)+i×sin(4П/5).
Как правильно возвести это всё в 5-ю степень, может кто-нибудь расписать?
394120
#268 #393405
>>393392
скобки можно расставить:
(g_1g_2...g_n-2)g_n-1)g_ng_n+1)...)
Я честно как и >>393393 не знаю, как это доказать.
#269 #393412
Похоже, ко-ко-консрурктивист пошел стирать свои обосранные штанишки. Впрочем, ничего нового.
#270 #393413
>>393371

> На одном формализме до сих пор бы не доказали что 1+1=2. Построение - это определение объекта, я уже сто раз объяснял почему.


Ты ебанашка необучаемая, иди читай учебники блядь! Формализм это желание максимально подробно описать математику формальным языком. Самые ярые формалисты: Бурбаки и создатели HoTT. Противники формализма: Брауэр, который поэтому даже не формализовал интуиционистскую логику, пришлось другим за него это делать. Что ты несешь то блядь?

Далее, Гентцен использовал трансфинитную индукцию. Какой конструктивизм блядь? Что ты несешь? Можно ли представить что-то более невероятное из области бесконечного, чем трансфинитную индукцию?
393423
#271 #393414
>>393387

>определение N через нумералы Черча.


Где-то есть тред Computer Science, тут же занимаются математикой, а не компьютерной поеботой.
393425
#272 #393423
>>393413

>Самые ярые формалисты: Бурбаки


На самом деле нет, они не формалисты. Для них эстетика всегда важнее формальности.
393429
#273 #393425
>>393414
Ты так говоришь, как будто современная математика - это не компьютерная поебота.
393426
#274 #393426
>>393425
Что ты подрузумиваешь под современной?
393427
#275 #393427
>>393426
Решение квадратных уравнений перебором, очевидно же.
393428
264 Кб, 858x662
#276 #393428
>>393427
https://vk.com/mathsecret
Выброшенный Богом не палится.
#277 #393429
>>393423
В принципе да. Компьютерные формалисты зашли куда дальше.
90 Кб, 241x300
#278 #393435
>>393383

>А ты видишь разницу между математическим объектом и фотонами и бозонами? Какие могут быть математические правила построения у элементарных частиц, совсем уже школота тутошняя поехала.


Математика - часть физики.
-В.А. Арнольд
393438
14 Кб, 252x349
#279 #393438
>>393435
Натуральные числа изобрёл бог Тот.
-В.И. Арнольд
393440
#280 #393440
>>393438
Думаю, это он в переносном смысле.
393441
#281 #393441
>>393440
В прямом.
393442
#282 #393442
>>393441
А ссылку дашь? Интересно же почитать такого выдающиегося тролля.
393443
#283 #393443
>>393442
Он не троллил. Он катался на велосипеде и его переехал грузовик.
393444
#284 #393444
>>393443
Я слышал это уже. Что насчёт ссылки.
393446
#285 #393446
>>393444
Гугли сам.
393447
#286 #393447
>>393446
А ты вредный.
#287 #393465
>>393396
всё произведение вырождается в три сомножителя, среди которых мы можем ставить скобки как угодно, не?
Вообще скучное доказательство и сама теорема, но хочется разобраться, Винберг потому что сказал "легко видеть"
#288 #393485
Доказал гипотезу Римана. Где опубликовать работу, чтобы получить свой миллион? Даже Arxiv.org не пускает, говорит, одобрение нужно.
Доказательство подробное и лёгкое, поймёт любой, знакомый с комплексным анализом.
393487393495
#289 #393487
>>393485
1. Идёшь на bitcoin brain wallet generator.
2. walletgenerator.net
3. Генеришь биткоин адрес. (Туда же можешь запросить и эти свои лиммионы)
4. Сохраняешь приватный (СЕКРЕТНЫЙ) ключ у себя.
5. Вбиваешь биткоин адрес в http://wallet-2sx53n.sakurity.com/#sign
6. Вместо message - своё доказательство.
7. Нажав кнопку sign message подписываешь сообщение приватным ключем.
8. Проверить цифровую подпись можно здесь: http://wallet-2sx53n.sakurity.com/#verify - в результате, будет адрес.
9. Т. к. никто, кроме тебя не знает приватный ключ - никто не сможет сгенерировать новое сообщение, подписанное твоим адресом.
Ведь сообщение подписывается приватным ключом.

Подписанное сообщение выглядит примерно так:

-----BEGIN BITCOIN SIGNED MESSAGE-----
This is an example of a signed message.
-----BEGIN SIGNATURE-----
1HZwkjkeaoZfTSaJxDw6aKkxp45agDiEzN
G464uQHxQKFoM5REYcTJN6JOd4Ezzg2bDY7E7VIa0ICwRf/CS9hs9f/F3i/SF7b5of6le6+BWVaUnqE3wyAjvL0=
-----END BITCOIN SIGNED MESSAGE-----

Можешь также, юзать трипкоды.
https://2ch.hk/dr/res/97.html (М) , но это так, для общего развития...
#290 #393488
>>393487
10. Постишь подписанное сообщение здесь.
11. И можешь ссылаться на него, даже когда тред уйдёт в бамплимит.
12. Твоё доказательство с цифровой подписью наверняка будет на архиваче.
(http://arhivach.org/)
Там же, сможешь найти и этот тред по ссылке, и свой пост.
13. Не забудь вставить дату в подписанное сообщение. Она как-бы докажет, что ты опубликовал это раньше.
14. ?????
15. PROFIT!
393491393512
#291 #393491
>>393487
>>393488
Алсо, brainwallet генератор можешь слить в zip-архиве, в виде JS и HTML-кода и открыть его у себя в браузере с диска. Он работает локально, и не требует сетевого подключения, по сути.

https://github.com/MichaelMure/WalletGenerator.net/archive/master.zip
Вот он, на github'e.
393493393512
#292 #393493
>>393491
А вот тебе bitcoin signature tool, на JS, которая тоже работает локально.
https://github.com/ReinProject/bitcoin-signature-tool/archive/master.zip
393512
#293 #393495
>>393485

>чтобы получить свой


Вообще-то обещают выплату Математический институт Клэя, значит - тебе туда.
Но доказательство - лучше всё-равно подпиши. И даже время с микросекундами вставь. =)
393498
#294 #393497
>>393487
Ещё раз:

1. Допустим, walletgenerator.net, при генерации, выдал:
5JdYN7qdXEFnC6hrbPCQV9UdE4jX6Zvj2YWfAx9Lo4dzGrso71c - приватный ключ. Знаешь только ты.
1CcdZSAVDvHBaZdmZygWEKcKd6zZyY3eSt - биткоин адрес.

2. Вбиваешь всё это http://wallet-2sx53n.sakurity.com/#sign или же в bitcoin signature tool:
Private key: 5KYZdUEo39z3FPrtuX2QbbwGnNP5zTd7yyr2SC1j299sBCnWjss
address - получается из приватного ключа, но не наоборот.
message
"Доказательство Васи Пупкина.
Дата: Wednesday, October 19, 2016
Время: 13:41:37

Курлык-курлык-курлык."
Нажимаешь "sign message" -> получаешь подписанное сообщение:
"-----BEGIN BITCOIN SIGNED MESSAGE-----
Доказательство Васи Пупкина.
Дата: Wednesday, October 19, 2016
Время: 13:41:37

Курлык-курлык-курлык.
-----BEGIN SIGNATURE-----
1HZwkjkeaoZfTSaJxDw6aKkxp45agDiEzN
GyJbLY3gaq8sRsVKI1AguEoptP8qY6vVuRL9FndVLbWuYUGVrY/AhSf8s3NjFPOS1Dhqlg0pBBLI4OFxHp8CeJw=
-----END BITCOIN SIGNED MESSAGE-----"

Вот его и постишь везде и всюду, и во все поля,
пока институт Клея - сам к тебе не прийдёт.
Можешь запостить и здесь своё доказательство [spoielr]и никто его не спиздит, потому что подписанное[/spoiler], а мы замутим "одобрение" - нас легион.
#294 #393497
>>393487
Ещё раз:

1. Допустим, walletgenerator.net, при генерации, выдал:
5JdYN7qdXEFnC6hrbPCQV9UdE4jX6Zvj2YWfAx9Lo4dzGrso71c - приватный ключ. Знаешь только ты.
1CcdZSAVDvHBaZdmZygWEKcKd6zZyY3eSt - биткоин адрес.

2. Вбиваешь всё это http://wallet-2sx53n.sakurity.com/#sign или же в bitcoin signature tool:
Private key: 5KYZdUEo39z3FPrtuX2QbbwGnNP5zTd7yyr2SC1j299sBCnWjss
address - получается из приватного ключа, но не наоборот.
message
"Доказательство Васи Пупкина.
Дата: Wednesday, October 19, 2016
Время: 13:41:37

Курлык-курлык-курлык."
Нажимаешь "sign message" -> получаешь подписанное сообщение:
"-----BEGIN BITCOIN SIGNED MESSAGE-----
Доказательство Васи Пупкина.
Дата: Wednesday, October 19, 2016
Время: 13:41:37

Курлык-курлык-курлык.
-----BEGIN SIGNATURE-----
1HZwkjkeaoZfTSaJxDw6aKkxp45agDiEzN
GyJbLY3gaq8sRsVKI1AguEoptP8qY6vVuRL9FndVLbWuYUGVrY/AhSf8s3NjFPOS1Dhqlg0pBBLI4OFxHp8CeJw=
-----END BITCOIN SIGNED MESSAGE-----"

Вот его и постишь везде и всюду, и во все поля,
пока институт Клея - сам к тебе не прийдёт.
Можешь запостить и здесь своё доказательство [spoielr]и никто его не спиздит, потому что подписанное[/spoiler], а мы замутим "одобрение" - нас легион.
393512
#295 #393498
>>393495
Спасибо, конечно, за такой надёжный способ подтверждения авторства, но если я опубликую док-во сюда, то как Математический Институт Клэя сможет о нём узнать?
Здесь вообще есть хоть пара ананонов, кто знает на среднем уровне комплексный анализ? Или только множество N обсуждать горазды?

У них в правилах написано, что перед вручением премии научное сообщество моё док-во должно проверить и признать. Для этого нужна публикация в журнале, а не на окраине интернета. Наверное.
Вот Перельман свои потоки Риччи на arxiv.org опубликовал, у меня даже на архиве нет разрешения на публикацию.
#296 #393499
>>393498
Миша Вербицкий может посмотреть, если аноны одобрят, кто-нибудь скинет ссылку на тифаретчик.
#297 #393501
>>393498

>научное сообщество моё док-во должно проверить и признать


Ну так, чтобы это было, надо ж распиарить это дело значит - в среде научного сообщества, разумеется.
Чё ты сидишь-то, а?

Подпиши и вайпай треды, лол.

Можешь подписанное сообщение почтой на бумажке в институт Клея отправить.
Вот ихний адрес: Clay Mathematics Institute
70 Main St
Suite 300
Peterborough, NH 03458
USA
CMI President’s Office
Andrew Wiles Building
Radcliffe Observatory Quarter
Woodstock Road
Oxford OX2 6GG, UK

Можешь его на билборды прицепить, на форумы всякие запостить, чаты заспамить и т. д.
393502
#298 #393502
>>393501
На википедию прицепить, в конце-концов...
#299 #393505
>>393498
кидай своё доказательство.
вангую ошибку там найти сможет и второкурсник, лол
43 Кб, 645x239
#300 #393507
Аноны! Нид хелп!
Решал за бабосы контрольную, препод поставил за задание минус. кто из нас в глаза ебётся? не вижу ошибки в упор
#301 #393510
>>393498
У них что, Интернета нет?
Сослаться на пост можешь с помощью
символа #
https://2ch.hk/sci/res/392366.html#393498 (М) , где
393498 - номер поста.

Или, например, вот так: http://arhivach.org/thread/206244/#393264
на архиваче, когда тред уйдёт в бамплимит или его выпилят.
#302 #393512
>>393487
>>393488
>>393491
>>393493
>>393497
>>393498

Да, и вот ещё... Подпись нужна для проверки того, что сообщение отправлено именно от адресата, а не от какого-то рандом анона.
Адрес - открытая информация (хеш публичного ключа), приватный ключ - закрытая инфа.

Текст - открытая инфа.
Т. к. рандом анон может извлечь инфо и подписать её другим ключём, с другим адресом в результате проверки подписи, позже причём,
значит адрес надо опубликовать прежде чем будет опубликовано сообщение содержащее цифровую подпись.

Такие дела.
393517
#303 #393517
>>393512
Ты с тифаретника что ли?

мимо
393518393527
#304 #393518
>>393517
Не, с чего бы это?
393519
#305 #393519
>>393518
Ты попутал что ли, лох? Ты нахуй на моём тифаретнике?
393521393524
#306 #393521
>>393519
Это ты попутал. Пшёл отсюда.
393522
#307 #393522
>>393521
Хуй соси, губой тряси.
393523
#308 #393523
>>393522
Ну давай, покажи как ты обычно это делаешь.
393525
#309 #393524
>>393519

>на моём тифаретнике


Вербит, что-ли?
#310 #393525
>>393523
Пепка прыгни
393529
#311 #393527
>>393517

>Ты с тифаретника что ли?


>Ты нахуй на моём тифаретнике


Хрень какая-то без https и SSL. Двощи рулят.

>Пепка прыгни


Аж туда прокинул этим?
393548
#312 #393529
>>393525
А там, чувак "доказал Гипотезу римана".
Повыше.
393548
#313 #393538
>>393498
Вербит говорил о какой-то альтернативе архиву для ноунейм бомжей. Правда я не помню, как называется
393542
#314 #393539
Ещё раз, давайте поможем найти мне где ошибка в доказательстве, поскольку в кострикине доказательство больше по объёму, а значит моё неправильное:
Верно ли следующее доказательство обобщённого закона ассоциативности?
Пусть G - группа.
Требуется доказать, что произведение g1 g2 ... gn не зависит от расстановки скобок.
Док-во:
по индукции. база n=3 - верно (аксиома)
предположим, что верно для всех n меньших k,
докажем, что верно для n = k
(g1 g2 ... g_k-1)gk = ((g1 g2...gm)(gm+1...g_k-1))g_k
Пусть g1 g2...gm = a
gm+1...g_k-1 = b
Тогда (ab)g_k = a(b*g_k) в силу аксиомы
т.е. наше произведение из n cомножителей выродилось в произведение трёх сомножителей, среди которых мы можем ставить скобки как угодно. Т.е. мы доказали, что результат не зависит от расстановки скобок, а значит обобщённый закон ассоциативности доказан.
бля я понимаю, что это пиздецки скучная теорема и доказательство чисто техническое и без идеи, но мне надо его усвоить, чтобы двигаться дальше.
алсо вторую задачу из учебника винберга (доказать что любая подгруппа Z имеет вид nZ ) я спокойно решил
393541
#315 #393541
>>393539
Забей на это гавно, блядь.
И ты доказал, что g1g2g3...gn+1 = (((g1g2g3...gm)(gm+1gm+2...)g_k)
А нужно для любой расстановки скобок.
К примеру для (g1
g2g3...)(gk1gk2gk3...*gk) ты нихуя не доказал. Хотя, возможно, я тупой дебил несу хуйню
393544
#316 #393542
>>393538
vixra.org
#317 #393544
>>393541
я такой аутист, что пока с этим говном не разберусь дальше не смогу нормально книгу читать.
и я доказал, что g1...gk = ((g1...g_m)(g_m+1...g_k-1))g_k = (g1...g_m)((g_m+1...g_k-1)g_k)
бля. теперь я понял где соснул.
393545393546393569
#318 #393545
>>393544
какого хуя винберг говорит тогда "нетрудно вывести". ну да, для него нетрудно наверное. петухан бля
#319 #393546
>>393544

>я такой аутист, что пока с этим говном не разберусь дальше не смогу нормально книгу читать.


Избавься от этой привычки, она очень сильно тормозит и не факт, что ты решишь задачу.
#320 #393548
>>393527
>>393529
Пепка, прыгни. Молодец. Лови кость.

Ну и болото тут у вас. Особенно рассмеялся с хуилы выше, писавшего, что ДОКАЗАЛ ГИПОТИЗУ РИМАНА КУДА АПУБЛИКОВАТЬ ПАМАГИТЕ.
Просто гнилое болото. На dxdy вас бы всех обоссали и перебанили, особенно "доказавшего" Римана - его бы ещё и выебли.
Умрите.
393552393571
#321 #393552
>>393548

>На dxdy вас бы всех обоссали и перебанили, особенно "доказавшего" Римана - его бы ещё и выебли.


Дед, плис, обоссали бы может да, у половины дедов там энурез, а вот что выебали не факт, чирикалка в таком приклоном возрасте просто не работает.
393555
#322 #393555
>>393552
Соси хуй.
То есть, ты защищаешь "доказавшего" Римана? правильно понял, петушок?
393556393571
#323 #393556
>>393555

>Соси хуй.


Твоя мать-шлюха будет его сосать.

>То есть, ты защищаешь "доказавшего" Римана? правильно понял, петушок?


Дед, в твоём возрасте делать логичные выводы сложно и я действительно сочувствую тебе. Но я имел в виду, что таких поехавших, как решишвий задчу, на dxdy полно, один Мунин чего стоит с его тензорами, которые придумал Эйнштейн. И никто его не обоссывает и не ебёт, говорю же у дедов чирикалка давно не работает.
393559
#324 #393559
>>393556
Сука, тебя, выродка, нужно назад в дыру матери твоей впихнуть
http://dxdy.ru/topic3476.html
393571
#325 #393569
>>393544
Я на это и намекал здесь (>>393396). Тебе нужно доказать, что (g_1 × g_2 × … × g_m) × (g_{m+1} × … × g_n) = (g_1 × g_2 × … × g_l) × (g_{l+1} × … × g_n). Вот тогда ты докажешь, что скобки можно расставлять как угодно. Чтобы это доказать, удобно воспользоваться левонормированной формой произведения. Короче, ничего сложного у Кострикина нет, но пишет он запутанно.
#326 #393571
>>393559
Дед, ты обосрался иди смени памперс, просто представь сколько правил dxdy ты нарушил этими постами:
>>393548
>>393555
>>393559
Так, что обоссали бы именно тебя там а не решателя гипотезы римана.
393619
#327 #393588
Можно ли представить любую алгебраическую формулу как таковую с целыми коэффициентами или это в учебнике хуёво написали?
393589
#328 #393589
>>393588
Нет, в учебники хуёво написали.
#329 #393590
Можно ли скипнуть этот ваш анализ и сразу перейти к годноте вроде теории категорий, топологии, теории узлов, графов и прочей годноте? А то читаю и какая-то хуйня серая, вообще не интересно.
#330 #393591
>>393590
Конечно, можешь учить всё сразу.
#331 #393603
>>393590
Теория категорий абстрактна, но перед ней лучше изучить мотивировку этих всех обозначений, т.е. алгебру.
Топология, теория узлов как ее часть, теория графов, они вообще довольно независимы, применяются в других местах, но сами посторонних знаний не требуют.
393612
#332 #393606
>>393590
Можно, но зачем?.. Они все равно не конструктивны.
393607
#333 #393607
>>393606

>не конструктивны


Фотон построил уже?
#334 #393610
>>393590
Есть Aluffi, Algebra Chapter 0. Там алгебра (которая пригодится по-любому) вводится на языке категорий, отличная вещь. Пререквизитов никаких не надо. Имхо, чтобы учить теорию категорий с ходу нужно быть невообразимым аутистом - слишком абстрактно - мотивации и применения будут непоняты как мне кажется.
393612
#335 #393612
>>393603
>>393610
Вы аутята, вас спросили не о теории категорий, а о том, стоит ли читать анализ.
393613
#336 #393613
>>393612

>вас спросили не о теории категорий



>Можно ли скипнуть этот ваш анализ и сразу перейти к годноте вроде теории категорий ...


Я ответил на эту часть - что да, можно, но через алгебру.
А предыдущий анон вообще один в один на поставленный вопрос ответил полностью.
#337 #393619
>>393571

>а не решателя гипотезы римана


А вдруг он её не решил, а просто толстит?
393653393656
#338 #393653
>>393619
А вдруг он её решил?
#339 #393656
>>393619
Это не важно, важно то, что тото деда обоссали бы на dxdy энурезные деды.
393658
#340 #393658
>>393656
*того
#341 #393675
>>393590
Теоретико-категорное обобщение понятия функции это пучок. Функции работают с числами, пучки с векторными пространствами. Анализ нужно учить на языке пучков. Начни с понятия векторного расслоения.
Основное понятие в алгебре это модуль. Модули бывают над кольцом, так что начни с изучения колец. Группы (не абелевы) не нужны, это отдельная область. Кольцо это абелева группа по сложению + ассоциативное умножение + дистрибутивность. Модуль это обобщение понятий абелевой группы и векторного пространства. Скаляры в векторном пространстве образуют поле, для модуля достаточно кольца. Модуль над кольцом, который сам является кольцом, называется алгеброй.
Векторное расслоение это обобщение понятия векторного пространства. Если слои в векторном расслоении являются алгебрами, то оно называется когеретным пучком, но это в алгебраической геометрии.
Пучок бывает над топологическим пространством. Группа сечений пучка определена для каждого открытого подмножества пространства, плюс отображение ограничения для каждого включения. Локально тривиальное векторное расслоение (fibre bundle) локально гомеоморфно декартову произведению, пучок же локально гомеоморфен самому пространству. Пучок колец это способ сопоставить гомоморфизмы колец включениям открытых множеств. Конверсия глобальных данных в локальные.

Теория категорий и ее мотивация лежат в гомологической алгебре. И гомологическая алгебра, и теория представлений, и алгебраическая к-теория изучают модули. В последней области модули нужны проективные, поскольку hom functor is exact они дают локально свободный пучок.
Про анализ пусть расскажет кто-то другой, мне он нахуй не нужен лично. Или читай Касивара, Шапира.
393683393711
#342 #393681
>>392366 (OP)
Матаны, нужен учебник/пособие по механике сплошных сред, жидкость. И задачник к нему. На дхду ничего не нашел, выручайте.
#343 #393682
Раз речь зашла о функциях. Существуют ли неконструктивные определения функции, т.е. такие, которые не дают возможность построить функцию (сопоставить элементы из области определения элементам из области значений)?
393688393937393941
#344 #393683
>>393675
Осмелюсь спросить, это ты открыватель реальностей из 11 треда?
393695393705
#345 #393685
Кстати, что круче, все книги Элементов математики Бурбаки или программа Вербицкого?
393687393705
#346 #393687
>>393685
По количеству интересного хардкора - программа Вербицкого, с большим запасом. По количеству фолловеров - Бурбаки.
#347 #393688
>>393682

>неконструктивные


Выше уже всё обсуждали.
#348 #393695
>>393683
Этот открыватель знатно обосрался, говоря, что R не нужно.
393705
#349 #393705
>>393683
Верно, это я.
>>393695
Обосрался, по следующей причине: один человек заверил меня, что полнота по дедекинду и архимедовость это эквивалетные требования. А про теорему Островского я просто забыл. Что касается R, то оно и правда не нужно, мне по крайней мере.
>>393685
Программа Вербицкого достаточно хороша для самостоятельного изучения, по причине разообразия тем. Схематически её можно представить так:
2 курс: спектралки, хар классы, коммутативная алгебра.
3-й: топологическая к-теория, гомотопическая алгебра, алгебраическая геометрия
4-й: алгебраическая к-теория, этальные когомологии
5-й: программа мори, гипотезы вейля.
Плюс процентов 30 того, что интересно Михаилу Сергеевичу.
Когомологии Галуа уже можно заменить мотивными когомологиями им. Воеводского. Каледин мог бы в этом манифесте написать про циклические гомологии, поскольку у него нет такой боли, как у тифарета (утверждающего что когомологии де рама проще сингулярных и вообще топологическая мотивация не нужна).
Этальные когомологии на четвертом курсе нужны для последующего изучения работы Делиня. По мнению Вербицкого геометрический ленглендс это и есть современная математика.
Эллиптические когомологии, топологические модулярные формы и вообще производную математику оба дружно отрицают, тут лучше слушать более адекватных людей.
Что интересно в имеющемся списке -- последовательное изучение сначала топологической к-теории, потом алгебраической к-теории. Общего у них только К0. Топологическая это про операторные С* алгебры, алгебраическая же раздел линейной алгебры. Это экстремизм, это хорошо и правильно. В этом смысле его же программа для ВШЭ уныла и не интересна, поскольку экстремизма уже нет, а вкусовщина и неосиляторство остались.
393707393708393728
#350 #393707
>>393705
Ты всё ещё считаешь, что математик должен быть мистиком и эзотериком?
Ко-ко-консурктивист нас тут верунами и мистиками обзывает, как ты к этому относишься?
393714
#351 #393708
>>393705
Заходи сюда почаще, я скучаю. Ждал твоего возвращения с 11! треда.
393710
#352 #393710
>>393708

>11!


>!


>11!=39916800


Полегче со знаками восклицания, мужик. Навряд ли столько мат-тредов вообще будет.
393712393720
#353 #393711
Последний абзац получился скомканным, поясню. С алгебры это что-то на пересечении функционального анализа, теории представлений и некоммутативной геометрии. Линейная алгебра же по определению наука* о модулях, см >>393675 пост. Для топол. К-теории нужна переодичность Ботта (см. "Теорию Морса" Милнора), для алгебраической не нужна.
Алгебраическая крайне сложна и интересна -- BGL(R)+ Квиллена, категория Вальдхаузена, те же мотивы появляются отсюда.
А вот теория представлений в контексте геометрического Ленглендса это ультрамейнстрим, каждый желающий попасть в Гарвард обязан этим заниматься, перефразируя Арнольда можно говорить о гельфандской мафии в московской школе, находясь вне которой можно быть только очередным маргиналом с нлаба.
#354 #393712
>>393710
Не верно, да, но это типа ударение на слове.
393713
#355 #393713
>>393712
Я понял, что ты хотел сказать.
#356 #393714
>>393707
Конструктивисту кстати можно было бы форсить Вайлдбергера, отрицающего слишком большие числа и не употребляющего R вообще. Серьезно, у него есть только rational, decimal и complex. Это охуенно, по моему. Общая топология и то, что считается основами анализа, достаточно легко вытекают из теории множеств Кантора и достаточно нахуй никому не нужные за её пределами.
Отношусь так: Кантор спиздил все у более гениального Дедекинда (который еще и первым ввел определение N понятие мат. индукции, дал определение поля, кольца и идеала, алгебраически построил римановы поверхности и тд). А Брауэр, например, за один семинар сформировал Витгенштейна-2 как философа. После написания трактата тот разочаровался в позитивизме и считал философию бесполезным занятием.
#358 #393717
>>393714
Ну наш конструктивист отрицает матиндукцию и аксиому выбора в принципе, заодно с исключенным третьим, так что и топология у него pointless, и анализ конструктивный, что маргинальщина невероятная.
393723
#359 #393718
>>393714
В смысле про Витгенштейна? Расскажи про него. Он применял математическую нить в своей философии языка?
393719
#360 #393719
>>393718
Конечно же нет, математика у него на таком же иллюстративном уровне, как и у Канта. А философские исследования он написал через 20 лет после злополучной лекции.
393721393723
#361 #393720
>>393710
Что двачеры будут обсуждать в математическом треде под номером 39916800? Конструктивиста?
393726
#362 #393721
>>393719
А во время войны он в дневнике писал, что типа потерял математическую нить своего философского рассуждения. Врал, собака?
#363 #393723
>>393717
Отрицание аксиомы выбора не такая уж и редкость, почитай блог Павлова, например. Да и сам Вербит говорит, что многие важные результаты можно получить и без нее, см его лекцию про Банаха-Тарского.
>>393719
После лекции он написал синюю и коричневую книги, из которых выросли философские исследование, при жизни так и не опубликованные. А математика его была на уровне Фреге и Рассела-Уайтхеда, потому что с ними он общался.
393725
#364 #393724
>>393714
Эй, я вилдербергера форсю, а конструктивист - петух и ненастоящий конструктивист. Но вы все еще хуже, фанатики ебаные.
#366 #393726
>>393720
Домашку с интегралами и производной.
Будут плеваться от того, что мы называли современной математикой и первой культурой. А нас будут называть дедами.
#367 #393728
>>393705
А после всей этой программы от Вербицкого можно начать понимать современные работы в алгебраической геометрии и теории чисел (те, что на arxiv.org) и работать с ними? А после книг Бурбаки, для сравнения?

Сам я по глупости инженегор четвёртого курса. Хотя всегда хотел стать математиком (первокультурным, а не интеграло-обезьяной, как сейчас).
393730
#368 #393730
>>393728
Да, и гораздо раньше. По Бурбаки нельзя, там алгебраической геометрии нет. Зато второй том по группам Ли хороший.
393744
#369 #393744
>>393730
Так, а евклидова геометрия нужна? У него в программе есть. Вообще, весь этот первый его блок "Матшкольник (школьная программа)" мне, нешкольнику, нужен? Не говорю, что знаю хотя бы половину тем в нём, просто он оторван от программы и эти вещи далее, вполне вероятно, прояснятся в предложенных им же учебниках не на школьном, а уже нормальном уровне.
Вот программа сама, если что: http://imperium.lenin.ru/~verbit/MATH/programma.html
393746393748
#370 #393746
>>393744
Евклидовой, аффинной и проективной геометрий достаточно в объеме книги Прасолова и Тихомирова. Про ортогональную и симлектическую можно читать у Артина (геометрическая алгебра) или Коксетера (введение в геометрию).
393748
#371 #393748
>>393744
>>393746
В 2004 в НМУ Миша с Димой вели спецпоток, где учили тому, что нужно было выучить в школе. Большинство тем из программы соответствуют тому, что дано в листочках(нет анализа, логики и ТМ).
http://ium.mccme.ru/f04/experimental.html
Здесь можешь pdf-ки скачать, но не все в pdf
http://imperium.lenin.ru/~verbit/MATH/LISTKI/TRIVIUM/
Конвертер из ps в pdf
https://online2pdf.com/convert-ps-to-pdf
Есть опечатки в некоторых местах, но ты их заметишь, скорее всего.
393749
#372 #393749
>>393748
имхо это все бесполезно. Когда это объясняет лектор, потом приходишь обсуждать задачки с другими ребятами, решения проверяют, то это годнота, а в отрыве от людей нет.
393750393751393769
#373 #393750
>>393749
Dead s dxdy, pliz
#374 #393751
>>393749
Можно на дваче листочки обсуждать и решать.
393752
#375 #393752
>>393751
Я, кстати, с твоей мамкой прорешал пару листочков вчера.
393753
#376 #393753
>>393752
И как успехи?
393754
#377 #393754
>>393753
с одночленами у нее все хорошо, а вот в многочленах она уже путается
393756
#378 #393755
Может кто-нибудь рассказать мне, как устроена аспирантура/докторантура, вот это вот всё (в области математики/компьютер саенс)?
Скажем, закончил человек магистратуру, идёт в аспирантуру, учится там три года. Что дальше?
И как эти степени транслируются в американские стандарты? В phd что переводится?
393762
#379 #393756
>>393754
Трехчлены брать умеет? Как интегралы берёт?
393758
#380 #393758
>>393756
Интегралы берет глубоко и вдумчиво, с пониманием сути дела.
393759
#381 #393759
>>393758
А с топлогиями что?
393760
#382 #393760
>>393759
Топлогии большие, объемные, приятные на ощупь, немного потнве.
393766
#383 #393762
>>393755

>закончил человек магистратуру, идёт в аспирантуру, учится там три года.


И master degree и phd получают в graduate school. Магистр для индустрии, phd для академии. Поступают туда бакалавры. На phd учатся пять лет, в первые два из которых ты проходишь обязательные курсы, а далее работаешь над тезисом.
В рф магистр обязателен, сделано это чтобы не убить специалитет в местах типа мехмата мгу. И магистры и специалисты могут поступить в аспирантуру и получить там phd -- кандидата наук. Дальше можно через какое-то время защитить докторскую. У нас это называется доктор наук, в Германии -- габилитация.
393763
#384 #393763
>>393762

>Дальше можно через какое-то время защитить докторскую.


То-есть, там нихуя не надо делать, кроме диссертации?
393765
#385 #393765
>>393763
Основные научные результаты докторской диссертации должны быть опубликованы в ведущих рецензируемых научных журналах и изданиях, перечень которых определяет ВАК.
#386 #393766
>>393760
Спасибо, что помогаешь моей мама с математикой.
Кстати, пока вы с ней занимались математикой, мы с твоей мамкой занимались сексом. Я ебал её во все щели, засовывал ей свой хуец в очко, а потом она слизывали с его говно. Ещё она облизала мне анус. Твоя мам развратная женщина, я такого, что она вытроряет даже в порно не видел.
#387 #393767
>>393714
Ты не ответил про мистику и эзотерику.
393770
6 Кб, 364x150
#388 #393769
>>393749
Мне не с кем и я не понимаю зачем их обсуждать? Если твоё решение тебя удовлетворяет, то мне кажется этого достаточно.
393774
#389 #393770
>>393767
Обвинение в мистицизме было довольно привычным делом для канторовской теории, которая в итоге рухнула под грузом парадоксов. Воскресить ее не удалось и более поздние математики решили притвориться, что так и надо.
Источник таких обвинений -- вы берете логику, справедливую в отношении финитных объектов, и необоснованно начинаете применять ее к инфинитным.
Под эзотерикой и мистикой я понимал не это, а отход от рационализма прежде всего.
393773393780
#390 #393773
>>393770
Ясно.
#391 #393774
>>393769
Ну лично я так много неверных и неточных доказательств сочинил, шаги какие-то пропустил, важные проверки. А еще товарищ может придумать более изящное доказательство и за какую-то очень сложную теорему можно взяться вдовем.
#392 #393780
>>393770

>берете логику, справедливую в отношении финитных объектов, и необоснованно начинаете применять ее к инфинитным


Покажи несколько примеров такого.
393784
#393 #393784
>>393780
Существование недостижимого кардинала. Первая проблема Гильберта. Вообще любое бесконечное множество.
Classical logic was abstracted from the mathematics of finite sets and their subsets …. Forgetful of this limited origin, one afterwards mistook that logic for something above and prior to all mathematics, and finally applied it, without justification, to the mathematics of infinite sets. This is the Fall and original sin of [Cantor's] set theory.
393792
#394 #393792
>>393784
Это список buzzword'ов. А ты покажи, пожалуйста, именно то, как математики "берут логику, справедливую в отношении финитных объектов, и начинают применять её к инфинитным". На конкретных примерах. Желательно без больших кардиналов, так как для их содержательного изучения необходимы аксиомы, отличающиеся от канона ZFC, и я уверен, что с этими аксиомами в этом треде никто не работает.
#395 #393794
Я действительно рад, что на дваче есть не раздел, но хотя бы треды математические.

поункукаре-кун
393795
#396 #393795
>>393794
Я специально купил пасскод и отправил заявку на /math, но ответа нет. С Абу свяжитесь, если хотите, попросите обезьяну мою заявку одобрить.
393800
#397 #393800
>>393795
непроеби, Саш
#398 #393827
вопрос на миллион: как доказать, что прямая и плоскость не могут пересекаться по отрезку?
393833393835
#399 #393833
>>393827
Если две точки прямой принадлежат плоскости, то и вся прямая принадлежит плоскости.
393836
#400 #393835
>>393827
Через две точки можно провести одну и только одну прямую.
Это, кажется, аксиома принадлежности, из Начал Евклида.
393836
#401 #393836
>>393833
>>393835
Если прямая лежит на плоскости, значит она пересекается с ней в точках этого конкретного отрезка, не?
393839
#402 #393839
>>393836
Существуют точки прямой, не лежащие в этом отрезке, но принадлежащие пересечению.
393842
23 Кб, 569x422
#403 #393842
>>393839
Ну, пускай у тебя будет неевклидова геометрия
и кривое пространство с неким коэффициентом кривизны.
И пускай в этом пространстве будет плоскость.
Ты сможешь нарисовать прямую, удовлетворяющую твоим условиям?
393843
11 Кб, 500x387
#404 #393843
>>393842
Или даже не прямую, а кривую, потому что пространство кривое же.
#405 #393848
>>392869
В этих лекциях было: http://ium.mccme.ru/f15/algebra-3.html

Задачи там же и, например, в Атье-Макдональде, с. 47.
Ещё в книжке Прасолова есть про Tor_1 (там он назван просто функтором Tor).
Ещё - английская Википедия и какие-то книги и записки лекций, найденные по запросу "tor/ext functor" и "group cohomology".
Внятного последовательного изложения в книгах, увы, не знаю.

А ответ на свой вопрос нашёл здесь, 10 страница: http://www.mi.ras.ru/~kazarian/papers/chcl06.pdf
127 Кб, 452x460
#407 #393898
Поясните, если я НИХУЯ НЕ ПОНИМАЮ ВООБЩЕ что делать, хотя теорию вроде читал, я даун необучаемый? Может лучше забить на анализ и в алгебру углубляться.
393899394132
#408 #393899
>>393898
А это случием не ты оскорблял меня вышел?
393908
#409 #393908
>>393899

> случием


Ты пидар, что ли? Обиженка какая? Так соси хуй, не отвлекайся.
Ответь только.
393909
#410 #393909
>>393908
Ах ты лживый хуесос, статья у него по функану. А сам-то даже в интегралах не может разобраться, вот же питух. И хуй дед твой сосёт.
54 Кб, 1026x409
#411 #393937
Мой вопрос >>393682 так и остался без внятного ответа, поэтому посмотрим на вещи еще шире. С какой стати вы вообще считаете ZFC чем-то неконструктивным, если оно легко и непринужденно строится в пруверах? https://coq.inria.fr/V8.2pl1/contribs/ZFC.html И с какой стати считать аксиомы ZFC именно аксиомами, а не конструктивными правилами построения?
#412 #393940
>>393937
Ты у ко-ко-консруктививиста местного спрости, когда он штанишки постирает, то сюда придёт.
#413 #393941
>>393937
>>393682
Ну можно, скажем, взять функцию, которая сопоставляет числу его колмогоровскую сложность (т.е. длину кратчайшей программы, выписывающей это число). Никакого эффективного метода вычислить это нет.
393942394016
#414 #393942
>>393941
Это пример функции. А не пример определения функции как объекта/явления вообще.
#415 #393964
>>393937
Булеан неконструктивен.
Аксиома выбора неконструктивна.
394016394117
#416 #393986
Как же на выходных тут тухло.
393988
#417 #393988
>>393986
Давай я в какой-нибудь аксиоме усомнюсь и назову тебя веруном.
393999
#418 #393999
>>393988
Давай, а я тебе ко-ко-консруктивистом. Как в старые-добрые времена!
394011
#419 #394011
>>393999
Для разнообразия аксиома подстановки. Какой наркоман её придумал? Типа, если есть множество M и есть функциональное высказывание f, в которое можно подставлять множества, - то f-образ M снова множество, как?! Что за бред, объясняйтесь веруны. С фига ли у вас можно преобразовать одно множество в другое, просто указав правило преобразования элементов?
1066 Кб, Webm
#420 #394016
>>393964
webm

>>393941
А на какомязыке эта прогамма написана?
Вот можешь пример самый тривиальный и нетривиальный, но очень простой. Ну или книгу норм, счего там начинать надо.
#421 #394077
Посоветуйте учебник по абстрактной алгебре. Чтобы замкнутое и более или менее современное изложение (если не с категориями, то хотя бы с теорией множеств изначально).
Желательно на английском. Их много, не знаю, что выбрать.
Спасибо.
394125
#422 #394096
Стоит ли читать второй том Зорича, кто-то из анонов говорил, что не надо.
394109
#423 #394109
>>394096
Анон плохого не посоветует.
394110
#424 #394110
>>394109
Это сарказм?
394111
#425 #394111
>>394110
Да, это сарказм. Второй том зорича на голову полезнее первого. Как минимум интереснее.
394112
#426 #394112
>>394111
Спасибо. Пойду читать.
#427 #394117
>>393964
Только в случае неконструктивных множеств. Я же сто раз давал ссылку на реализацию ZFC через зависимые типы. Все там конструктивно, если не основано на вере в бесконечность саму по себе, вне хотя бы и потенциально построимого объекта.
394187
#428 #394120
>>393403
>>393403
Формула Муавра
394128
#429 #394125
>>394077
Algebra chapter 0 буквально пару постов назад советовали на аналогичный вопрос.
#430 #394128
>>394120
Хочешь сказать, что
(cos(2П4/5) + i×sin(8П/5))5 =
15×(cos(5×2П4/5) + i×sin(5×8П/5)) =
15×(cos(8П) + i×sin(8П))?
А дальше что?
#431 #394132
>>393898
Пиздец, с кем я на борде сижу. Просто посмотри определение интеграла простой функции, тогда, если не последний даун, поймешь, как это доказывается. Там и доказывать то нечего.
5 Кб, 407x71
#432 #394138
>>392366 (OP)
Помогите доказать эту хуиту методом математической индукции.
394145
#433 #394145
>>394138
Ты лох, что ли?
394149394151394231
#434 #394149
>>394145
Нет. Я тополог, а та картофан!
#435 #394151
>>394145
Да. Поэтому и прошу помощи.
#436 #394152
Аноны, т.к. шапка у вас говно, буду постить:
как закатиться в алгебру "для себя"с возможными профитами? Знание нулевоену складывать вычитать да делить могу, но допустим таблицу умножения так себе знаю. Откуда начать? Даст ли школьная программа что-то полезное? Пните в нужном направлении, как освою прибегу дальше спрашивать.
394153394154394155
#437 #394153
>>394152
Алгебра Гельфанд Шень, Гашков Элементарная алгебра, Алексеев теорема Абеля в задачах и решениях, рандомный учебник по алгебре.
#438 #394154
>>394152
Понял, чтопроебался. Зачем несколько тредов для математики?
#439 #394155
>>394152
Спасибо!
#440 #394187
>>394117
Так как я я не распознал, буквально каким формализмом ты пользуешься в >>393937, я не берусь судить, что ты понимаешь под "реализацией", где именно ты её проводишь и нет ли у тебя ошибок. Но хочу обратить внимание на то, чем это быть не может. Как несложно показать, ZFC доказывает непротиворечивость базовой теории типов Мартин-Лёфа, как впрочем и ряда её вариаций с аксиома вселенных (кстати, относительно недавно было доказано, что это имеет место даже если добавить аксиому унивалентности). Но если бы твоя "реализация" сводила бы вопрос о непротиворечивости ZFC к непротиворечивости одной из перечисленных выше теорий, то ZFC доказывала бы свою непротиворечивость и была бы противоречива в силу второй теоремы Гёделя о неполноте. Вообще говоря, нет никаких причин полагать, что какие-либо разумные с концептуальной точки зрения теории зависимых типов окажутся сильнее ZFC или хотя бы, что они будут непротиворечивы, но это нельзя будет установить в ZFC (см. https://www1.maths.leeds.ac.uk/~rathjen/typeOHIO.pdf ). Тем самым здесь не может выходить сведение непротиворечивости, а тем самым довольно странно называть это реализацией.
394188394222
#441 #394188
#442 #394190
правильно я понимаю, что для прикладных задач достаточно модульной арифметики, матриц и производных?
394191
#443 #394191
>>394190
Смотря для каких прикладных задач, для некоторых придёться самому подбирать разделы.
394192
#444 #394192
>>394191
ну для вычислений на компе например что может кроме перечисленного пригодиться?
394193
#445 #394193
>>394192
Опять же для вычислений чего? Гугли численные методы, для разных задач они разные.
#447 #394221
>>394217
Не нужно.
#448 #394222
>>394187

>нет никаких причин полагать, что какие-либо разумные с концептуальной точки зрения теории зависимых типов окажутся сильнее ZFC или хотя бы, что они будут непротиворечивы


Я же тут >>393144 3ий пик, привел тезис Мартин-Лёфа насчет того, почему конструктивная форма суждений полна в отличие от неконструктивной. За твою "концептуальную точку зрения" ничего не могу сказать, что ты там взял и откуда. С парадоксами и противоречиями в конструктивной математике все просто - посылки, ведущие к противоречиям просто выбрасываются как бесполезные, нарушающие вычислимость и все. Так было с парадоксом Жирара в MLTT.

>довольно странно называть это реализацией.


А что это https://coq.inria.fr/V8.2pl1/contribs/ZFC.html если не реализация? Этот Акцель, насколько мне известно, вообще предлагал конструктивный вариант ZFC использовать вместо MLTT, там целая книжка есть, я не читал правда.
394225
#449 #394223
>>394217
Там капча сломана, треды не создаются.
#450 #394225
>>394222

>>довольно странно называть это реализацией.


>А что это https://coq.inria.fr/V8.2pl1/contribs/ZFC.html если не реализация?


Честно говоря, здесь еще надо разбираться, что именно там реализовано - настоящая ZFC над классической логикой или тот или иной ограниченный вариант (скажем CZF в самом деле погружается в теорию типов). Кстати прямо в абстракте там указывается, что нужны неконструктивные аксиомы

> A non-computational type-theoretical axiom of choice is necessary to


prove the replacement schemata and the set-theoretical AC.
Так что может и правда здесь самая обычная ZFC, но не в базовом Calculus of Inductive Constructions, а в его расширение неконструктивными аксиомами. Но если это так, то конечно занятно взглянуть, чем именно является теоретико-типовая аксиома выбора и почему она столь сильно усиливает теорию, но заявлять, что ZFC конструктивно реализована полностью некорректно.

>За твою "концептуальную точку зрения" ничего не могу сказать, что ты там взял и откуда.


Еще раз, я привел вовсе не случайную ссылку (http://www1.maths.leeds.ac.uk/~rathjen/EHPanthology.pdf). Автор (кстати, крупный специалист по интуиционистским теориям множеств) довольно подробно анализирует возможные расширения MLTT, которые лежат в рамках изначального подхода. Он формулирует некоторую систему T и неформально аргументирует, что в неё погружаются всевозможные концептуально правильные потенциальные расширения MLTT (она сама уже лежит за рамками этого подхода), а далее оказывается, что по силе T находится на уровне некоторого фрагмента арифметики второго порядка (теорема 6.3), т.е. гораздо ниже ZFC.
394228
#451 #394228
>>394225

>настоящая ZFC над классической логикой или тот или иной ограниченный вариант (скажем CZF


А чем "настоящая" лучше? И вообще, не та ли настоящая, которую можно построить, а не веровать в нее?
394229
#452 #394229
>>394228

>И вообще, не та ли настоящая, которую можно построить, а не веровать в нее?


Пользуйся теми теориями которые тебе нравятся, никто ведь не мешает, но аббревиатура ZFC имеет общепринятое понимание и не нужно пытаться его подменить.

>А чем "настоящая" лучше?


Скажем, CZF и ZFC - это разные системы, решающие разные задачи. ZFC - это формализация неформальной канторовской теории множеств. Поэтому она прекрасно подходит (хотя зачастую и крайне избыточна) для формализации областей математики построенных в теоретико-множественном духе (т.е. большей части математики) и также для собственно изучения теории множеств. CZF - это попытка сохранить фрагмент теории множеств, оставаясь в рамках интуиционистской парадигмы.
394235
#453 #394231
>>394145
В /d/ зайди. Да, ты знаешь куда.
#454 #394235
>>394229

>CZF и ZFC - это разные системы, решающие разные задачи.


Ой, всё. А не разные подходы к одному и тому же, не?
394236394253
#455 #394236
>>394235
к чему конкретно?
394237
#456 #394237
>>394236
К основаниям.
99 Кб, 678x678
#457 #394238
Есть тут добрые умные аноны, которые могли бы мне за один вечер объяснить эту хуйню? Ебучую таблицу значений я почти выучил, а как всё это устроено не понял. Формулы какие-то ещё, блджад
394241394247
#458 #394241
#459 #394247
>>394238
гельфанд шень тригонометрия
394256
#460 #394253
>>394235

>А не разные подходы к одному и тому же, не?


Одна теория полноценная формализация канторовской теории множеств, другая нет (что там насчет диагонального метода в CZF? ах да, ведь в CZF даже нет нормальной аксиомы степени).
#461 #394256
>>394247
У меня время до утра, чтобы понять. Хуево напишу завтра - хуево пройдёт все полугодие.
394259
#462 #394259
>>394256
У тебя вся жизнь хуево пройдет, а ты о полугодии.
394261
#463 #394261
>>394259
Если мне кто-нибудь это объяснит, то, может быть, и нормально пройдёт.
394264
#464 #394262
Тут есть нмушники? Сколько листков вы уже сдали по геометрии?
#465 #394264
>>394261

>может


Не может.
394265
#466 #394265
>>394264
Хватит это. Лучше помоги мне
394266
#467 #394266
>>394265

>Лучше


Не лучше.
Просто умри и не мучайся зря.
394274
#468 #394274
>>394266
Ты какой-то ебанутый, ты знал?
394275
#469 #394275
>>394274
Да, я охуенен.
Конечно знал.
394279
#470 #394279
>>394275
Объясни тригонометрию, пидор.
394282
24 Кб, 812x278
#471 #394281
https://www.youtube.com/watch?v=ARt4_D0xnaw
Блять как же проигроваю с коментов этого опущенного репера.
#472 #394282
>>394279

>Объясни тригонометрию


Деньги вперед.
394286
#473 #394284
В /math наплыв ньюфагов (или троллей, но некоторые точно ньюфаги). Мне попросить убрать ссылку на /math из шапки?
#474 #394286
>>394282
У меня нет денег, я школьник.
394288
#475 #394288
>>394286

>я школьник.


Раздевайся.
#476 #394289
Как на бумаге посчитать значение арктангенса 2 радиан, к примеру?
394292
#477 #394292
>>394289
Школьник, угомонись.
62 Кб, 1200x979
#478 #394300
Все укатились в матдоску?
394303394305394307
#479 #394303
#480 #394305
>>394300
Алсо, классный пик.

Прошу ОПа треда математики для начинающих создать тред в /math.
#481 #394307
>>394300
Когда вы съебете наконец то можно будет создать нормальный тренд математики для нематематиков.
394313
#482 #394313
>>394307

>можно будет создать нормальный тренд


Нельзя, я туда насру.
#483 #394506
Новый положняк такой:
Тред для новичков перекатывается в /math.
Обычный мат-тред, остаётся в /sci и превращается в тред математики для не математиков, ну или филиал /math в /sci.
Перекатываемся =======> https://2ch.hk/math/res/200.html (М)
394507394513
# OP #484 #394507
#485 #394513
>>394506
Посмотрел эту новую доску, мда-мда, оп чсвшное мудило и пошел нахуй со своей доской в пизду.
394515394518
#486 #394515
>>394513
Дурачок? Где там ЧСВ?
# OP #487 #394517

>оп чсвшное мудило и пошел нахуй со своей доской в пизду.


Это ты про меня или про создателя доски?
394518394519
# OP #488 #394518
>>394517
>>394513

>оп чсвшное мудило и пошел нахуй со своей доской в пизду.

394519
#489 #394519
#490 #394527
5a+3a-a=(5+3-1)a=7a
Я так и не пойму зачем ставиться ко­эф­фи­ци­ент 1, когда нету числа рядом с буквою?
394532394536
#491 #394532
>>394527
Тся.
#492 #394536
>>394527
потому что цифры это многочлен, а буквы прост для удобства
#493 #394551

>http://youtu.be/dSi5bykReiQ


С 30й минуты, поясните поцаны, я понимаю что он доказывает на пальцах, но почему он использует в доказательстве windinw numbers? Разве не достаточно что у него между малым кругом вокруг проекции нуля и большим-большим кругом - непрерывная функция и она покроет собой область нуля? Причем здесь winding numbers?
Осторожно, в видео - сектанство.
#494 #394553
Аноны, помогите, залетаю к вам из программача. Надо написать функцию на JS, которая будет добавлять к пройденному пути курсором, тормозной путь. Нужна формула для вычисления этого тормозного пути у меня есть velocity - т.е скорость, есть начальная и конечная координата.
394555
#495 #394555
>>394553
Добавь к скорости отрицательное ускорение.
#496 #394571
>>392366 (OP)
Поясните за процесс синтеза молекул нуклеиновых кислот, гомологичных вирусному геному с точки зрения гомологий и когомологий.
394578
#497 #394578
>>394571
В математике другие гомологии и когомологии, чем в химии.
394582
#498 #394582
>>394578
Гомологии — одно из основных понятий алгебраической топологии.
Так что без топологий тоже не обойтись.

А чтоб вкатиться в когомологии, надо походу котопологии.
#499 #394598
Ну и что за говно? Из этого треда убежали все. На ту доску набежали школьники и засрали все треды домашками за восьмой класс. Какого хуя, а? По-моему стало хуже, не?
394599394600394619
#500 #394599
>>394598
Полностью с тобой согласен.
#501 #394600
>>394598
С мейлру все адекваты кроме меня давно съебали. Нет ничего интересного в общении со школуйней, чей предел - аргументация уровня "азаза мамку твою ебал, поссал на опущенца)))".
394601
#502 #394601
>>394600
А куда идти? Куда идти-то?
394602
#503 #394602
>>394601
На добрчан.
394603
#504 #394603
>>394602
Там ещё мертвей. За неделю около 5-ти постов.
394604
#505 #394604
>>394603
Потому что все по делу. У тебя получается написать больше пяти постов о математике в неделю?
394605
#506 #394605
>>394604
Конечно, я в день около 6-20 постов пишу по математике пишу.
394606
#507 #394606
>>394605
Например? Книгоёбство не считается.
394607
#508 #394607
>>394606
Как минимум эти посты в /math

>>273


>>295


>>240


>>278


>>267


>>276


>>282

394609
#509 #394609
>>394607
По делу только 278, остальное - флуд.
394610
#510 #394610
>>394609
С чего это друг? Или ты представляешь беседу в мат-треде в виде онли решения задачи школьникам?
394611
#511 #394611
>>394610
Ну уж всяко не в виде смехуёчков про Перельмана.
394612
#512 #394612
>>394611
Тут ты тоже ошибаешся, смехуечки нужны.
394613
#513 #394613
>>394612
Нет. Нужны только разговоры по делу. Для смехуёчков есть раковники впаше.
394615394616
#514 #394615
>>394613
Нет, просто смехуечки неотъемлимая часть мат-треда. А сейчас ты просто пытаешься оправдать мёртвость доброчана тем, что там якобы говорят только по делу. Если бы там было больше анонов, то и смехуечки были бы.
394617
#515 #394616
>>394613
Гротендик нассал на тебя только что.
#516 #394617
>>394615
Почему же "якобы"?
394618
#517 #394618
>>394617

>Если бы там было больше анонов, то и смехуечки были бы.

#518 #394619
>>394598
Не ври. На всём /math поста четыре с домашней из трёхсот. Модератор /math может удалять и такие редкие просьбы о школьной домашней, но вряд ли это нужно, если ты не ебанутый.
/math сейчас - просто отображённый в доску мат. тред.
394622394629
#519 #394622
>>394619

>На всём /math поста четыре с домашней из трёхсот.


Да, остальные посты - в треде рыбникова или ℕ-петуха.
394623
#520 #394623
>>394622
Там только 34 поста. Причём, это нафлудили два анона(один из которых я). И да Рыбников-тред как раз и создан ради того, чтобы там можно было бы оттянутся и мальца пошитпостить.
#521 #394629
>>394619
Я не вру. А ты не оправдывайся. Гоните нахуй школьников оттуда (хотя это все равно не поможет).
#522 #394652
Насколько далеко простирается множество Мандельброта? Его "ветви" конечны или любая область на плоскости заселена бесконечным множеством этих жопастых букашек?
394654
#523 #394654
>>394652
Второй вариант.
#524 #394727
>>392366 (OP)
Вопрос по теории вероятностей, и комбинаторике.
Пусть есть число 42.
Задача - найти количество способов его разложения на различные множители.
0 Кб, 168x18
#525 #394781
>>392366 (OP)
помогите сделать задание по функциональному анализу. Есть пространство l∞, и есть условие:
∀k∈N существует предел числовой последовательности x_n(k).
Необходимо ответить на вопросы, является ли заданное условие
a) необходимым;
б) достаточным;
в)необходимым и достаточным
для сходимости последовательности x_n в метрическом пространстве X?
Объясните пожалуйста, как сделать это задание?
84 Кб, 1280x719
#526 #394867
>>392401
Анон, я сегодня охуел, когда на паре посчитали функцию Римана для минус единицы и получили ответ - 1/12, меня просто как молнией ударило. Одно дело, когда ты читаешь это как какой-то рандомный факт, а другое дело - когда логически выводишь и понимаешь, от чего у Чебышёва взрывался мозг, и испытываешь похожие ощущения. Математика - ты замечательная.
12 Кб, 575x54
#527 #394879
Понятно, что для любой композиции сдвига и ротации будет одна неподвижная точка, но если от этого еще и гомотетию меньше 1 взять (или вообще любую кроме 1), то каким нахуй образом эта точка останется неподвижной, если только специально её центром гомотетии не взять (но это противоречит условию ЛЮБАЯ композиция).

Вроде на стаке поясняли, но линалом. А тут без линала должна решаться задача, так что нахуй то говнорешение.

Альзо как доказать биноманальную формулу ньютана индукцией без всякой комбинаторики?
394951
#528 #394951
>>394879
Иди нахуй и прекрати спамить этой хуйнёй.
395034
#529 #395034
>>394951
Хуй соси, говно. Если не можешь буквально доматан решить, тебе здесь нехуй делать. По-крайней мере, назад в ро съеби, говнарь.
395063
#530 #395063
>>395034
Хуеплёт, тебе уже три раза всё сказали, а ты продолжаешь выёбываться.

>Если не можешь буквально доматан решить, тебе здесь нехуй делать.


Когда решишь, тогда и приходи. А пока ты хуя собачьего не стоишь.
216 Кб, 384x1823
#531 #395301
В соседнем треде, который для начинающий, одним из комментов на пикрелейтед было "хватит форсить эту хуйню...На 666 можно поставить труды мочисуки и подобное (помниться тифарет когда-то перчислял, если найду выложу)"

Так и не выложил, хотя я его просил. Может кто-то восполнить этот пробел?
395327395475
#532 #395327
>>395301
Нахрена нужны гомологии, когомологии, какая от них реальная польза?
395385
#533 #395385
>>395327

> польза


Ты ошибся тредом, тебе к инженерам.
395389395540
#534 #395389
>>395385
Хуйло, математики просто обязанны быть полезными, быть полезнее инжинеров.
395391
#535 #395391
>>395389

> обязанны


Ок. Теперь уходи.
395392
#536 #395392
>>395391
Почему математик не должен приносить пользу людям?
395409
#537 #395409
>>395392
Потому что ему похуй на это.
#538 #395475
>>395301
на мой вопрос никто не ответит?
395481
#539 #395481
>>395475
В /math
#540 #395540
>>395385
Математика используется в астрономии, компьютерах, в играх, статистика и тд.
395591
#541 #395591
>>395540
И что?
# OP #542 #396078
Тред утонул или удален.
Это копия, сохраненная 15 февраля 2017 года.

Скачать тред: только с превью, с превью и прикрепленными файлами.
Второй вариант может долго скачиваться. Файлы будут только в живых или недавно утонувших тредах. Подробнее

Если вам полезен архив М.Двача, пожертвуйте на оплату сервера.
« /sci/В начало тредаВеб-версияНастройки
/a//b//mu//s//vg/Все доски